Download as pdf or txt
Download as pdf or txt
You are on page 1of 46

Excel Professional Services Inc.

Management Firm of Professional Review and Training Center (PRTC)


Online • Manila • Cavite • Laguna • Cebu • Cagayan De Oro • Davao
Since 1977

Management Services (MS) TRINIDAD/ALENTON/URO


FIRST PRE-BOARD EXAMINATION CPA Review February 19, 20 & 21, 2023

1. Madtack Company's beginning and ending inventories a. Flexible budget.


for the month of November are: b. Zero-based budget.
c. Static budget.
Nov. 1 Nov. 30 d. Strategic budget.
Direct materials P 67,000 P 62,000
Work-in-process 145,000 171,000 Use the following information for the next two questions.
Finished goods 85,000 78,000 Nanjones Company manufactures a line of products
distributed nationally through wholesalers. Presented below
Production data for the month of November follows. are planned manufacturing data for 2023 and actual data
Direct labor, P200,000 for November 2023. The company applies overhead based
Actual factory overhead, 132,000 on planned machine hours using a predetermined annual
Direct materials purchased, 163,000 rate.
Transportation in, 4,000
Purchase returns and allowances, 2,000 2023 Planning Date
Annual November
Madtack uses one factory overhead control account and Fixed overhead P1,200,000 P100,000
charges factory overhead to production at 70 percent of Variable overhead 2,400,000 220,000
direct labor cost. The company does not formally Direct labor hours 48,000 4,000
recognize over/underapplied overhead until year-end. Machine hours 240,000 22,000

Madtack Company's net charge to factory overhead Data for November 2023
control for the month of November is: Direct labor hours (actual) 4,200
a. P8,000 debit, overapplied. Direct labor hours (plan based on 4,000
b. P8,000 debit, underapplied. output)
c. P8,000 credit, overapplied. Machine hours (actual) 21,600
d. P8,000 credit, underapplied. Machine hours (plan based on output) 21,000
Fixed overhead P101,000
2. Which of the following terms represents the residual Variable overhead P214,000
income that remains after the cost of all capital,
including equity capital, has been deducted? 6. The variable overhead spending variance for November
a. Free cash flow. 2023 was:
b. Market value-added. a. P2,000 favorable. c. P6,000 unfavorable.
c. Economic value-added. b. P6,000 favorable. d. P2,000 unfavorable.
d. Net operating capital.
7. The fixed overhead volume variance for November
3. Spring Co. had two divisions, A and B. Division A 2023 was:
created Product X, which could be sold on the outside a. P1,200 unfavorable. c. P5,000 favorable.
market for P25 and used variable costs of P15. Division b. P5,000 unfavorable. d. P1,200 favorable.
B could take Product X and apply additional variable
costs of P40 to create Product Y, which could be sold for 8. Although there is no single format for the balanced
P100. Division B received a special order for a large scorecard, the report generally includes a variety of
amount of Product Y. If Division A were operating at full measurements associated with objectives classified by
capacity, which of the following prices should Division A critical success factors. Critical success factors often
charge Division B for the Product X needed to fill the include:
special order? a. Sales, net income, cash flow, and return on
a. P15 c. P25 investment performance.
b. P20 d. P40 b. Shareholder satisfaction, customer satisfaction,
vendor satisfaction and employee satisfaction
4. Superior Industries, sales budget shows quarterly sales issues.
for the next year as follows. c. Financial, internal business process, customer
Quarter Units and human resource considerations.
1 10,000 d. Throughput and lifecycle times.
2 8,000
3 12,000 9. Responsibility accounting defines an operating center
4 14,000 that is responsible for revenue and costs as a(n):
The company policy is to have a finished goods a. Profit center. c. Operating unit.
inventory at the end of each quarter equal to 20 b. Revenue center. d. Investment center.
percent of the next quarter's sales. Budgeted
production for the second quarter of the next year 10. Using regression analysis, Fairfield Co. graphed the
would be: relationship of its cheapest product line's sales with its
a. 7,200 units. c. 8,400 units. customers' income levels. If there is a strong statistical
b. 8,800 units. d. 10,400 units. relationship between the sales and customers' income
levels, which of the following numbers best represents
5. The use of standard costs in the budgeting process the correlation coefficient for this relationship?
signifies that an organization has most likely a. - 9.00 c. +0.93
implemented a: b. - 0.93 d. +9.00

Page 1 of 7 www.teamprtc.com.ph MS.1stPB5.23


TEAM PRTC

Using the absorption costing method, the peso value of


11. Which of the following best describes the impact of the company’s inventory on August 31 and the
increasing the sales price? absorption income, respectively, would be
a. The increase in sales price increases contribution Inventory Value Absorption
margin per unit, causing net income to decrease. Income
b. The increase in sales price increases contribution a. P27,000 P35,000
margin per unit causing the break-even point to b. P27,000 P 5,000
decrease. c. P36,000 P 5,000
c. The increase in sales price means more units are d. P36,000 P35,000
sold.
d. The increase in sales price means an increase in 16. Alpine Company wants to earn a 6% return on sales
variable cost. after taxes. The company's effective income tax rate is
40%, and its contribution margin is 30%. If Alpine has
12. The process by which a company's products or services fixed costs of P240,000, the amount of sales required
are measured relative to the best possible levels of to earn the desired rate of return is:
performance is known as: a. P400,000 c. P1,000,000
a. efficiency c. a standard costing system b. P1,200,000 d. P375,000
b. benchmarking d. variance analysis
17. Pluto Company plans to discontinue a department that
13. Planet Manufacturing Corporation is using the following has a contribution margin of P24,000 and P48,000 in
flexible-budget formula for annual indirect labor cost: fixed costs. Of the fixed costs, P21,000 can be avoided.
The effect of this discontinuance on Pluto’s overall net
Total cost = P12,000 +P0.75 per machine hour operating income would be a(an)
a. decrease of P3,000 c. decrease of P24,000
For the month of June, the operating budget is based b. increase of P3,000 d. increase of P24,000
upon 10,000 hours of planned machine time. Indirect
labor costs included in this planning budget are 18. Green Company’s unit cost of manufacturing and selling
a. P7,500 c. P17,500 a given item at an activity level of 10,000 units per
b. P8,500 d. P19,500 month are:
Manufacturing costs
14. Simple regression differs from multiple regression in Direct materials P24
that: Direct labor 8
a. multiple regression uses all available data to
estimate the cost function, whereas simple Variable overhead 5
regression only uses simple data Fixed overhead 6
b. simple regression is limited to the use of only the Selling expenses
dependent variables and multiple regression can Variable 11
use both dependent and independent variables Fixed 8
c. simple regression uses only one independent
variable and multiple regression uses more than
The company has an inventory of 3,000 of this item
one independent variable
left over from last year’s model. These must be sold
d. simple regression uses only one dependent variable
through regular channels at reduced prices. The
and multiple regression uses more than one
inventory will be valueless unless sold this way. What
dependent variable
unit cost is relevant for establishing the minimum
selling price of these 3,000 units?
15. Apo Company’s variable costing income statement for
a. P11 c. P48
August appears below:
b. P37 d. P62
Sales (P15 per unit) P600,000
Less variable costs:
19. Information on Uranus’ direct material costs for May is
Variable cost of goods sold: as follows:
Beginning inventory P 72,000
Add variable cost of goods 315,000 Actual quantity of direct materials 30,000 lbs.
manufactured purchased and used
Goods available for sale 387,000 Actual cost of direct materials P84,000
Less ending inventory 27,000 Unfavorable direct materials usage P 3,000
Variable cost of goods sold 360,000 variance
Standard quantity of direct materials 29,000 lbs
Variable selling expenses 80,000
allowed for May production
Total variable costs 440,000
Contribution margin 160,000 For the month of May, Uranus’ direct materials price
Fixed costs: variance was:
Fixed manufacturing P105,000 a. P2,800 favorable c. P6,000 unfavorable
Fixed selling and administrative 35,000 b. P2,800 unfavorable d. P6,000 favorable
Total fixed costs 140,000
Net income P 20,000 20. Joker Company employs a standard absorption system
for product costing. The standard cost of its product is
The company produces 35,000 units each month. as follows:
Variable production costs per unit and total fixed costs Direct materials P14.50
have remained constant over the past several months. Direct labor (2 direct labor hours at P8) 16.00
Manufacturing overhead (2 DLH at P11) 22.00

Page 2 of 7 www.teamprtc.com.ph MS.1stPB5.23


TEAM PRTC

The manufacturing overhead rate is based upon a he watched the most recent batch of five balloons
normal activity level of 600,000 direct labor hours. floating into space, he thought surely, there must be
Joker planned to produce 25,000 units each month some way to avoid this waste.
during the year. The budgeted annual manufacturing
overhead is: Gibo has the opportunity to rent a helium canister that
Variable P3,600,000 would enable him to fill balloons while working in the
Fixed 3,000,000 park. Renting the canister would cost P80 per day.
Deflated balloons can be purchased at a price of P10.00
During November, Joker produced 26,000 units. each.
Joker used 53,500 direct labor hours in November at
a cost of P433,350. Actual manufacturing overhead What is the estimated Gibo’s profit if the helium canister
for the month was P250,000 fixed and P325,000 is rented assuming there are 5 balloons being unsold?
variable. a. No effect
b. Increase the profit by P150
The manufacturing overhead controllable variance for c. Increase the profit by P90
November is d. Decrease the profit by P80
a. P13,000 unfavorable c. P3,000 favorable
b. P10,000 favorable d. P4,000 favorable Use the following information for the next two questions.

21. The purchasing manager at Cara Company knows the Underfoot Products uses standard costing. The following
amount of raw materials required for each unit of the information about overhead was generated during May:
single product the company manufactures. Which of the
following statements best describes the information the Standard variable overhead rate P2 per machine hour
purchasing manager will require to estimate the Standard fixed overhead rate P1 per machine hour
quantity of raw materials to purchase?
Actual variable overhead costs P381,000
a. The amount of raw materials on hand at the
beginning of the period and the minimum quantity Actual fixed overhead costs P175,000
of raw materials management wishes to have in Budgeted fixed overhead costs P190,000
inventory at the end of the period Standard machine hours per unit 10
b. The amount of raw materials on hand at the produced
beginning of the period, the minimum quantity of Good units produced 18,000
raw materials management wishes to have in Actual machine hours 200,000
inventory at the end of the period, and the
estimated quantity of product the company expects
25. Compute the fixed overhead budget variance.
to sell in the period
a. P5,000 (F) c. P10,000 (F)
c. The amount of raw materials on hand at the
b. P5,000 (U) d. P15,000 (F)
beginning of the period, the minimum quantity of
raw materials management wishes to have in
26. Compute the fixed overhead volume variance using
inventory at the end of the period, and the
variable costing system.
estimated quantity of product the company expects
a. P5,000 (U) c. P10,000 (F)
to produce in the period
b. P10,000 (U) d. P0
d. The amount of raw materials required to meet the
estimated quantity of product the company expects
27. ______ refers to avoiding competition in making a
to produce in the period
product distinct from that of competitors by adding
value or features for which consumers are willing to pay
22. In multiproduct situations, when sales mix shifts toward
more.
the product with the highest contribution margin then:
a. Kaizen c. Confrontation
a. total revenues will decrease
b. Product differentiation d. Cost differentiation
b. breakeven quantity will increase
c. total contribution margin will decrease
28. A company would be reducing its discretionary costs if
d. operating income will increase
it
a. fired a production supervisor.
23. Hanlon Company has recorded the following data
b. closed its research and development department.
regarding its inventories. To manufacture product
c. successfully negotiated a reduction in its factory
MPC1, it takes 4 hours to move raw materials to the
rent.
work-in-process area. Transforming the raw materials
d. reduced its direct labor costs by hiring temporary
into finished goods takes three production steps of 3
workers.
hours each. The finished goods are then temporarily
stored in a storage area for 10 hours. Finally, they are
29. Which of the following factors would suggest a switch
inspected which takes 4 hours. Calculate the
to activity-based costing?
manufacturing cycle efficiency for product MPC1.
a. Product lines similar in volume and manufacturing
a. 52.94% c. 14.29%
complexity.
b. 48.15% d. 33.33%
b. Overhead costs constitute a significant portion of
total costs.
24. Gibo Zamora sells helium balloons in Luneta Park on
c. The manufacturing process has been stable.
Saturdays as a means of making some extra spending
d. Production managers use data provided by the
money. He buys balloons from a party store at a cost
existing system.
of P12 each and sells them at a price of P30.00 each.
Demand for the balloons ranges between 35 and 45 per
30. SuperClose Company follows and aggressive financing
day. Gibo normally purchases 40 balloons. The helium
policy in its working capital management while
loses its resilience in a couple of days, so balloons not
SuperSweet Corporation follows a conservative
sold cannot be saved for the following Saturday. Gibo
financing policy. Which one of the following statements
usually just releases the balloons that do not sell. As
is correct?

Page 3 of 7 www.teamprtc.com.ph MS.1stPB5.23


TEAM PRTC

a. SuperClose has low ratio of short-term debt to total then how many units did the company produce during
debt while SuperSweet has a high ratio of short- March?
term debt to total debt. a. 12,000 units c. 11,000 units
b. SuperClose has a low current ratio while b. 18,000 units d. 19,000 units
SuperSweet has a high current ratio
c. SuperClose has less liquidity risk while SuperSweet 37. Budgets set at very high levels of performance (i.e.,
has more liquidity risk. very low costs)
d. SuperClose finances short-term assets with long- a. assist in planning the operations of the company.
term debt while SuperSweet finances short-term b. stimulate people to perform better than they
assets with short-term debt. ordinarily would.
c. are helpful in evaluating the performance of
31. The sales mix for Emory's Hardware is as follows: managers.
Product A: 12 units @ P5.25 sales price; P4.85 variable d. can lead to low levels of performance.
cost per unit.
Product B: 10 units @ P7.50 sales price; P6.95 variable 38. Typically, managers have the LEAST control over the:
cost per unit. a. direct material price variance
Product C: 6 units @ P12.25 sales price; P10.35 b. direct materials efficiency variance
variable cost per unit. c. machine maintenance
Emory's fixed costs are P75,950. d. scheduling of production

What are the composite break-even units? 39. Halo Co. is currently operating at a loss of P15,000. The
a. 98,000 sales manager has received a special order for 5,000
b. 2,000 units of product, which normally sells for P35 per unit.
c. 3,500 Costs associated with the product are: direct material,
d. 4,000 P6; direct labor, P10; variable overhead, P3; applied
fixed overhead, P4; and variable selling expenses, P2.
32. A decrease in the price of a complementary good will The special order would allow the use of a slightly lower
a. shift the demand curve of the joint commodity to grade of direct material, thereby lowering the price per
the left unit by P1.50 and selling expenses would be decreased
b. increase the price paid for a substitute good by P1. If Halo Co wants this special order to increase
c. shift the supply curve of the joint commodity to the total net income for the firm to P25,000, what sales
the right price must be quoted for each of the 5,000 units?
d. shift the demand curve of the joint commodity to a. P18.50 c. P29.00
the right b. P24.50 d. P26.50

33. The local video store’s business increased by 12% 40. If the operating asset turnover ratio increased by 30
after the movie theater raised its prices from P65 to percent and the margin increased by 20 percent, the
P70. Thus, relative to movie theater admissions, divisional ROI
videos are a. would increase by 56 percent.
a. substitute goods c. complementary goods b. would decrease by 60 percent.
b. superior goods d. public goods c. would increase by 20 percent.
d. cannot be determined.
34. Alona Corporation has developed the following flexible
budget formula for monthly indirect labor costs: 41. The Tam Corporation makes a variety of leather goods.
Total Cost = P15,000 + P4.50 per machine hr. It uses standards costs and a flexible budget to aid
Operating budgets are based upon the monthly average planning and control. Budgeted variable overhead at a
of 10,000 machine hours of planned machine time. 45,000-direct labor hour level is P27,000.
Compute the flexible budget for indirect labor cost for
the year where production requires 115,000 machine During April material purchases were P241,900. Actual
hours. direct-labor costs incurred were P140,700. The direct-
a. P532,500 c. P690,000 labor usage variance was P5,100 unfavorable. The
b. P517,500 d. P697,500 actual average wage rate was P0.20 lower than the
average standard wage rate.
35. At the end of the accounting period Delas Alas
Corporation reports operating income of P30,000 and The company uses a variable overhead rate of 20% of
the fixed overhead cost rate is P20 per unit. Under standard direct-labor cost for flexible budgeting
variable costing, if this company produces 100 more purposes. Actual variable overhead for the month was
units of inventory, then operating income: P30,750.
a. will increase by P2,000
b. will increase by P2,000 only if the additional 100 What were the standard hours allowed during the
units of inventory are sold month of April?
c. will not be affected a. 51,950 c. 48,550
d. is indeterminable b. 56,925 d. 60,320

36. Laurel Company produces a single product. During 42. Quick Co. was analyzing variances for one of its
March, the company had net operating income under operations. The initial budget forecast production of
absorption costing that was P6,000 lower than under 20,000 units during the year with a variable
variable costing. The company sold 15,000 units in manufacturing overhead rate of P10 per unit. Quick
March, and its variable costs were P3 per unit, of which produced 19,000 units during the year. Actual variable
P3 was variable selling expense. If fixed manufacturing manufacturing costs were P210,000. What amount
overhead was P1.50 per unit under absorption costing would be Quick's flexible budget variance for the year?
and fixed selling & administrative expense was P0.50, a. P10,000 favorable.
b. P20,000 favorable.

Page 4 of 7 www.teamprtc.com.ph MS.1stPB5.23


TEAM PRTC

c. P10,000 unfavorable.
d. P20,000 unfavorable. 48. Which of the following best describes a firm's external
funding requirement?
43. Assuming costs are represented on the vertical axis and a. Growth in assets minus growth in liabilities minus
volume of activity on the horizontal axis, which of the net income
following costs would be represented by a line that is b. Growth in assets minus the current year's
parallel to the horizontal axis? retained earnings
a. total direct material costs c. Growth in assets minus growth in current
b. a consultant paid P75 per hour with a maximum liabilities minus net income
fee of P1,200 d. Growth in assets minus growth in current
c. employees who are paid P10 per hour and liabilities minus the year's retained earnings
guaranteed a minimum weekly wage of P200
d. rent on exhibit space at a convention Use the following information for the next two questions.

44. The coefficient of determination is Rogers Rods & Reels Ltd. manufactures and sells various
a. a measure of the variability of actual costs around types of fishing equipment. At the end of 2022, Rogers had
the cost-estimating equation. estimated for the production and sale of 15,000 bass fishing
b. used to construct probability intervals for cost rods. Each rod has a standard calling for 1.5 pounds of
estimates. direct material at a standard cost of P8.00 per pound and
c. a standardized measure of the degree to which 15 minutes of direct labor time at a standard cost of P.18
two variables move together. per minute. During 2023, Rogers actually produced and sold
d. a measure of the percent variation in the 16,000 rods. These 16,000 rods had an actual direct
dependent variable that is explained by an materials cost of P179,200 (25,600 pounds at P7.00 per
independent variable. pound) and an actual direct labor cost of P44,800 (224,000
minutes at P.20 per minute). Each rod sells for P50.
45. Donnelly Corporation manufactures and sells T-shirts
imprinted with college names and slogans. Last year, 49. What is Rogers' net income (loss) based on a flexible
the shirts sold for P7.50 each, and the variable cost to budget?
manufacture them was P2.25 per unit. The company a. P579,500 c. P576,000
needed to sell 20,000 shirts to break even. The net b. P564,800 d. P590,000
income last year was P5,040. Donnelly’s expectation
for the coming year include the following: 50. What is Rogers' flexible budget variance?
The sales price of the T-shirts will be P9 a. P11,200 F c. P 3,500 F
Variable cost to manufacture will increase by one-third b. P11,200 U d. P 3,500 U
Fixed costs will increase by 10%
The income tax rate of 40% will be unchanged 51. An approach to improvement that involves completely
redesigning business processes in order to eliminate
The selling price that would maintain the same unnecessary steps, reduce errors, and reduce costs.
contribution margin rate as last year is a. Total quality management
a. P9.00 c. P8.25 b. Process reengineering
b P10.00 d. P9.75 c. Theory of constraints
d. Life-cycle costing
46. Phelps Co. sells two products, Arks and Bins. Last year,
Phelps sold 12,000 units of Arks and 24,000 units of 52. Reporting under the direct costing concept is
Bins. Related data are: accomplished by
Product Unit Unit Unit a. Including only direct costs in the income statement.
Selling Variable Contribution b. Matching variable costs against revenues and
Price Cost Margin treating fixed costs as period costs.
Arks P120 P80 P40 c. Treating all costs as period costs.
Bins 80 60 20 d. Eliminating the work in process inventory
accounting
Assuming that last year's fixed costs totaled P910,000,
what was Phelps Co.'s composite break-even point? 53. Jones Company has P5,000,000 of average inventory
a. 34,125 c. 11,375 and cost of sales of P30,000,000. Using a 365-day year,
b. 27,302 d. 9,101 calculate the firm's inventory conversion period.
a. 30.25 days c. 45.00 days
47. Lord Industries manufactures a single product. Variable b. 60.83 days d. 72.44 days
production costs are P10 and fixed production costs are
P75,000. Lord uses a normal activity of 10,000 units to 54. Which one of the following statements concerning the
set its standard costs. Lord began the year with no capital intensity ratio is correct?
inventory, produced 11,000 units and sold 10,500 a. The capital intensity ratio is the amount of sales
units. The volume variance under each product costing generated from each peso of total assets.
are: b. The lower the capital intensity ratio, the greater the
Under Absorption Variable Costing amount of assets required to support each peso of
Costing sales.
a. P3,750 Unf P 0 c. A highly capital-intensive firm will have a low capital
b. P3,750 Fav P7,500 Unf intensity ratio.
d. The capital intensity ratio is the amount of total
c. P7,500 Unf P3,750 Unf
assets required to generate one peso of sales.
d. P7,500 Fav P 0
55. On a balanced scorecard, which of the following would
not be an example of a customer satisfaction measure?
a. Market share.

Page 5 of 7 www.teamprtc.com.ph MS.1stPB5.23


TEAM PRTC

b. Economic value added. Medium and 85% for Classy. Total fixed costs are
c. Response time. P225,000. If fixed costs will increase by 30 percent,
d. Customer retention. what amount of peso sales would be necessary to
generate an operating profit of P48,000?
56. Bidder’s budgeted sales for the coming year are a. P1,350,000 c. P1,135,000
P60,000,000, of which 80% are expected to be made b. P486,425 d. P910,000
on credit. Bidder wants to change its credit terms from
n/30 to 2/10 n/30 . If the new credit terms adopted, 63. ABC Company breaks even at P300,000 sales and earns
Bidder estimates that cash discounts would be taken on P30,000 at P350,000 sales. Which of the following is
40% of the credit sales and the uncollectible amount true?
would be unchanged. The adoption of the new credit a. Fixed costs are P20,000.
terms would result in expected discounts taken in the b. Profit at sales of P400,000 would be P80,000.
coming year of c. The selling price per unit is P3.
a. P 1,200,000 c. P 960,000 d. Contribution margin is 60% of sales.
b. P 384,000 d. P 480,000
64. Planners have determined that sales will increase by
57. Overland has a part with a unit cost of P40. It costs P52 25% next year, and that the profit margin will remain
to place, process and receive an order. The carrying at 15% of sales. Which of the following statements is
cost of the inventory is approximately 20% of the part's correct?
peso value per year. The firm uses 2,000 of the parts a. Profit will grow by 25%.
each year. What order quantity minimizes total b. The profit margin will grow by 15%.
inventory cost? c. Profit will grow proportionately faster than sales.
a. 93 c. 161 d. Ten percent of the increase in sales will become net
b.127 d. 205 income.

58. If D0 = P1.75, g (which is constant) = 3.6%, and P0 = Use the following information for the next two questions.
P32.00, what is the stock’s expected total return for the
coming year? Jordan Auto has developed the following production plan:
a. 8.37% c. 8.81%
b.8.59% d. 9.27% Month January February March April
Units 10,000 8,000 9,000 12,000
59. Abra, Inc., is contemplating a project that costs
P180,000. Expectations are that annual cash revenues Each unit contains 3 pounds of raw material. The desired
will be P70,000 and annual expenses (including raw material ending inventory each month is 120% of the
depreciation) will total P30,000. The project has a six- next month’s production, plus 500 pounds. (The beginning
year useful life and a residual value of P30,000. Assume inventory meets this requirement). Jordan has developed
Seattle Inc. uses straight line method of depreciation. the following direct labor standards for production of these
units.:
The accounting rate of return based on average
investment for the project is
Department 1 Department 2
a. 53.3% c. 22.2 %
b.66.7% d. 38.1% Hours per unit 2.0 0.5
Hourly rate P6.75 P12.00
60. Madden Company has projected its income before taxes
for next year as shown below. Madden is subject to a 65. How many raw material should Jordan Auto purchase in
40% income tax rate. March?
a. 27,000 pounds. c. 36,000 pounds.
Sales [160,000 units] P8,000,000 b. 32,900 pounds. d. 37,800 pounds.
Less: Cost of sales
Variable costs P2,000,000 66. Jordan Auto’s total budgeted direct labor pesos for
Fixed costs 3,000,000 5,000,000 February usage should be
Income before taxes P3,000,000 a. P156,000 c. P175,500
b. P165,750 d. P210,600
Madden’s net assets are P36,000,000. The peso
sales that must be achieved for Madden to earn a 67. ABC, Inc. is operating at full capacity with a sales level
10% after tax return on assets would be of P14M and fixed assets of
a. P8,800,000 c. P16,000,000 P7M. What is the required addition to fixed assets if
b.P12,000,000 d. P6,880,000 sales are to increase by 10 percent?
a. P350,000 c. P140,000
61. Sari-Sari Grocery is currently open only on Monday to b. P700,000 d. P280,000
Saturday. It is considering opening on Sundays. The
annual incremental costs of Sunday opening is 68. Gerald's Manufacturing is operating at 78 percent of its
estimated at P124,800. Its gross margin is 20%. It fixed asset capacity and has current sales of P575,000.
estimates that 60% of Sunday sales to customers would How fast can the firm grow before any new fixed assets
be on other days if its stores were not open on Sundays. are needed?
The Sunday sales that would be necessary for Sari-sari a. 22.00% c. 31.16%
to attain the same weekly operating income is b. 28.21% d. 37.00%
a. P19,500. c. P30,000.
b. P29,250. d. P20,000. 69. A firm's balance sheet as of December 31 is shown
below. The firm's sales for the year were
62. Gorio Co. provides two products, Medium and Classy. P1,000,000,000, and its after-tax margin on sales was
Medium accounts for 60 percent of total sales, variable 5%. Sales are expected to increase next year to
cost as a percentage of selling price are 60% for P1,300,000,000, and it plans to distribute 50% of its

Page 6 of 7 www.teamprtc.com.ph MS.1stPB5.23


TEAM PRTC

net profits to stockholders. Based on the percentage-


of-sales method, the amount of funds that must be
obtained externally by borrowing or by selling new
stock is

Assets (P millions) Liabilities (P millions)

Cash P 50 Accounts P 30
payable
Receivables 130 Accrued 40
taxes &
wages
Inventories 150 Mortgage 130
bonds
Net fixed 220 Common 150
assets stock
Retained 200
earnings
a. P111.50 million. c. P65 million.
b. P165 million. d. P144 million.

70. A hospital records the number of floral deliveries its


patients receive each day. For a one-week period, the
records show the following deliveries.

Day Number Deliveries


1 15
2 27
3 26
4 24
5 18
6 21
7 26

Using exponential smoothing with a smoothing


constant of 0.4 to forecast the number of deliveries,
calculate the forecast of deliveries in the third day.
Assume the forecast for day 1 are 15 deliveries.
a. 19.8 c. 26.4
b. 22.2 d. 26.6

End of Examination

Thank you for participating in Team PRTC


Nationwide Online Open First Pre-Board
Examination.

Page 7 of 7 www.teamprtc.com.ph MS.1stPB5.23


Excel Professional Services Inc.
Management Firm of Professional Review and Training Center (PRTC)
Online • Manila • Cavite • Laguna • Cebu • Cagayan De Oro • Davao
Since 1977

OCAMPO/OCAMPO/SOLIMAN/UY/
Auditing (AUD) CPA Review AGUILA/RICAFRENTE
FIRST PRE-BOARD EXAMINATION February 19, 20 & 21, 2023

Multiple Choice. Select the letter that corresponds to the these professionals is not required to present proof of
best answer. This examination consists of 70 items and the compliance with Continuing Professional Development
exam is good for three (3) hours. Good luck! (CPD) requirements?
a. Ella, a CPA, who works as a marketing specialist in
1. The objective of the attest function is to a big multinational firm. Her role does not require
a. Determine the validity of the transactions her to be a CPA.
examined. b. Rosanna, a staff in a small public accounting firm.
b. Express an opinion on the fairness of the financial She has been working with the firm for 5 years
statements. already.
c. Detect fraud. c. Vanessa, an entrepreneur with various business
d. Ascertain the consistent application of GAAP. interest related to food and beverage.
d. Kim, who passed the CPA examination 2 years ago.
2. An auditor should obtain sufficient knowledge of an She expects to renew her license for the first-time
entity's information system to understand the next year.
a. Safeguards used to limit access to computer
facilities. 8. Operational auditing is mainly concerned about:
b. Process used to prepare significant accounting a. Future improvements for management goals.
estimates. b. Past protection provided by current internal control.
c. Controls used to assure proper authorization of c. Verification of fair presentation of financial data.
transactions. d. Accuracy of data of financial records.
d. Controls used to detect the concealment of fraud.
9. Which of the following statements is correct concerning
3. Which of the following documents is not among the an auditor’s responsibility to report fraud?
requirements to be submitted when a candidate applies a. The auditor is required to communicate to the
for the CPA licensure examination with the Commission? client’s audit committee all minor fraudulent acts
a. A valid clearance from the National Bureau of perpetrated by low-level employees, even if the
Investigation (NBI). amounts involved are inconsequential.
b. Baccalaureate transcript of records with special b. The disclosure of material management fraud to
order if applicable. principal stockholders is required when both senior
c. Certificate of live birth in Philippine Statistics management and the board of directors fail to
Authority (PSA) security paper. acknowledge the fraudulent activities.
d. Form 138 Permanent Student Record. c. Fraudulent activities involving senior management
of which the auditor becomes aware should be
4. Individuals who commit fraud are ordinarily able to reported directly to the SEC.
rationalize the act and also have an d. The disclosure of fraudulent activities to parties
Incentive Opportunity other than the client’s senior management and its
a. Yes Yes audit committee is not ordinarily part of the
b. Yes No auditor’s responsibility.
c. No Yes
d. No No 10. One purpose of establishing quality control policies and
procedures for acceptance and continuance of client
relationships and specific engagements is to
5. Because of the risk of material misstatement, an audit
a. Undertake engagements only that the accounting
of financial statements in accordance with generally
firm is competent to perform.
accepted auditing standards should be planned and
b. Monitor significant deficiencies in the design and
performed with an attitude of
operation of the client’s internal control.
a. Objective judgment.
c. Identify noncompliance with aspects of contractual
b. Independent integrity.
agreements that affect the financial statements.
c. Professional skepticism.
d. Provide reasonable assurance that personnel will be
d. Impartial conservatism.
adequately trained to fulfill their assigned
responsibilities.
6. Which of the following procedures would an auditor least
likely perform while obtaining an understanding of a
11. Which one of the following is not a logical function of a
client in a financial statement audit?
CPA in public accounting practice?
a. Coordinating the assistance of entity personnel in
a. Attest function.
data preparation.
b. Supervision of internal audit staff.
b. Discussing matters that may affect the audit with
c. Tax practice.
firm personnel responsible for non-audit services to
d. Management consulting services.
the entity.
c. Selecting a sample of vendors' invoices for
12. When an auditor discovered that certain control
comparison to receiving reports.
activities were ineffective, the auditor most likely would
d. Reading the current year's interim financial
increase the
statements.
a. Level of detection risk.
b. Extent of tests of details.
7. The following professionals are planning to renew their
c. Level of inherent risk.
CPA ID card with the Commission soon. Who among
d. Extent of tests of controls.

Page 1 of 7 www.teamprtc.com.ph AUD.1stPB5.23


TEAM PRTC

13. The understanding with the client regarding a financial 19. Which of the following statements most likely would be
statement audit generally includes which of the included in an engagement letter from an auditor to a
following matters? client?
a. The expected opinion to be issued. a. The CPA firm will provide absolute assurance about
b. The responsibilities of the auditor. whether the financial statements are free of
c. The contingency fee structure. material misstatement.
d. The preliminary judgment about materiality. b. The CPA firm is responsible for ensuring that the
client complies with applicable laws.
14. The reliance placed on substantive tests in relation to c. The CPA firm will involve information technology
the reliance placed on internal control varies in a specialists in the performance of the audit.
relationship that is ordinarily d. The CPA firm will adjust the financial statements to
a. Parallel. correct misstatements before issuing a report.
b. Inverse.
c. Direct. 20. Which of the following comparisons would an auditor
d. Equal. most likely make in evaluating an entity's costs and
expenses?
15. Which of the following is the most reliable analytical a. The current year's accounts receivable with the prior
procedure to verify the year-end financial statement year's accounts receivable.
balances of a wholesale business? b. The current year's payroll expense with the prior
a. Verify depreciation expense by multiplying the year's payroll expense.
depreciable asset balances by one divided by the c. The budgeted current year's sales with the prior
depreciation rate. year's sales.
b. Verify commission expense by multiplying sales d. The budgeted current year's warranty expense with
revenue by the company’s standard commission the current year's contingent liabilities.
rate.
c. Verify interest expense, which includes imputed 21. In comparison to an external auditor, an internal auditor
interest, by multiplying long-term debt balances by is more likely to be concerned with
the year-end prevailing interest rate. a. Internal control.
d. Verify payroll tax liability by multiplying total payroll b. Cost accounting procedures.
costs by the contribution rate in effect during the c. Operational auditing.
year. d. Reviewing interim financial statements.

16. Which of the following is a step in an auditor's decision 22. Which of the following audit risk components may be
to assess control risk at below the maximum? assessed in nonquantitative terms?
a. Apply analytical procedures to both financial data Inherent risk Control risk Detection risk
and nonfinancial information to detect conditions a. Yes Yes No
that may indicate weak controls. b. Yes No Yes
b. Perform tests of details of transactions and account
c. No Yes Yes
balances to identify potential errors and
irregularities. d. Yes Yes Yes
c. Identify specific internal control policies and
procedures that are likely to detect or prevent 23. After obtaining an understanding of internal control and
material misstatements. assessing control risk of an entity, an auditor decided
d. Document that the additional audit effort to perform not to perform tests of controls. The auditor concludes
tests of controls exceeds the potential reduction in that
substantive testing. a. The available audit evidence obtained through tests
of controls would not support an increased level of
17. Assertions are representations by management that are control risk.
embodied in financial statements. Which of the b. A reduction in the assessed level of control risk is
following is not a management assertion? justified for certain financial statement assertions.
a. Obligations classified as long-term liabilities in the c. It would be inefficient to perform tests of controls
balance sheet will not mature within one year. that would result in a reduction in planned
b. Property is recorded at historical cost and such cost substantive tests.
is systematically allocated to appropriate accounting d. The assessed level of inherent risk exceeded the
periods. assessed level of control risk.
c. Finished goods inventory in the balance sheet are
available for sale. 24. Which of the following factors or conditions is an auditor
d. Net income reflects the earning power of the least likely to plan an audit to discover?
enterprise. a. Financial pressures affecting employees.
b. High turnover of senior management.
18. Which of the following are correct concerning the c. Inadequate monitoring of significant controls.
likelihood of loss and the potential amount involved with d. Inability to generate positive cash flows from
a material weakness? operations.
Likelihood of loss Potential amount involved
a. Probable More than inconsequential 25. In obtaining evidential matter in a regular audit, the
b. Probable Material auditor develops specific audit objectives and design
c. Reasonable possibility More than inconsequential substantive procedures relating to client’s assertions.
d. Reasonable possibility Material Accordingly:
1) The auditor obtains evidence as to management’s
assertions that all transactions and accounts that
should be presented are so included.

Page 2 of 7 www.teamprtc.com.ph AUD.1stPB5.23


TEAM PRTC

2) The auditor is concerned with management’s d. Whether or not the client reports to the Securities
assertions that it performed efficiently and and Exchange Commission.
effectively.
3) The auditor is concerned with management’s 32. If specific information comes to an auditor's attention
assertions that assets, liabilities, revenues and that implies noncompliance with laws that could result
expenses have been included at appropriate in a material, but indirect effect on the financial
amounts. statements, the auditor should next
4) The auditor obtains evidence as to client’s a. Apply audit procedures specifically directed to
assertions that the company can pay its obligations. ascertaining whether noncompliance has occurred.
a. All of the above statements are true. b. Seek the advice of an informed expert qualified to
b. Only statements (1) and (3) are true. practice law as to possible contingent liabilities.
c. Only statements (2) and (4) are true. c. Report the matter to an appropriate level of
d. All of the above statements are false. management at least one level above those
involved.
26. An auditor reviews a client’s accounting policies and d. Discuss the evidence with the client's audit
procedures when considering which of the following committee, or others with equivalent authority and
planning matters? responsibility.
a. Method of sampling to be used.
b. Preliminary judgments about materiality levels. 33. When performing a financial statement audit, auditors
c. Nature of reports to be rendered. are required to explicitly assess the risk of material
d. Understanding of the client’s operations and misstatement due to
business. a. Errors.
b. Fraud.
27. Before accepting an engagement to audit a new client, c. Illegal acts.
a CPA is required to obtain d. Business risk.
a. An understanding of the prospective client's
industry and business. 34. Which of the following is not represented in the Auditing
b. The prospective client's signature to the and Assurance Standards Council?
engagement letter. a. Association of CPAs in Public Practice.
c. A preliminary understanding of the prospective b. Insurance Commission.
client's control environment. c. Board of Accountancy.
d. The prospective client's consent to make inquiries of d. Bureau of Internal Revenue.
the predecessor auditor, if any.
35. Nerissa recently took the Licensure Examinations for
28. Matthews Corp. has changed from a system of recording CPAs and obtained the following rating: MS – 84.29%;
time worked with clock cards to a computerized AUD – 70%; TAX – 71.43%; RFBT – 83%; FAR –
timekeepingl system in which employees record time in 77.14% and AFAR – 78.57%. Which of the following
and out with magnetic cards. The computer system statements is correct:
automatically updates all payroll records. Because of a. Nerissa passed the CPA licensure examination.
this change b. Nerissa failed the CPA licensure examination.
a. A generalized computer audit program must be c. Nerissa needs to retake AUD and TAX within the
used. next two years from the preceding examination.
b. Part of the audit trail is altered. d. Nerissa needs to re-enroll in AUD and TAX related
c. The potential for payroll-related fraud is diminished. subjects in a CHED accredited school before she can
d. Transactions must be processed in batches. take another set of examination.

29. Which of the following is required documentation in an 36. The auditor's best defense when material
audit in accordance with generally accepted auditing misstatements are not uncovered is to have conducted
standards? the audit
a. A flowchart or narrative of the accounting system a. in accordance with generally accepted auditing
describing the recording and classification of standards.
transactions for financial reporting. b. as effectively as reasonably possible.
b. The overall audit strategy and audit plan. c. in a timely manner.
c. All major stakeholder groups. d. only after an adequate investigation of the
d. An internal control questionnaire identifying controls management team.
that assure specific objectives will be achieved.
37. Shown below (1 through 5) are the five types of tests
30. When an auditor considers a client’s internal control, which auditors use to determine whether financial
control activities ordinarily relate to performance statements are fairly stated. Which three are
reviews, information processing, segregation of duties substantive tests?
and 1. risk assessment procedures
a. Information and communication. 2. tests of controls
b. Operating decisions. 3. substantive tests of transactions
c. Physical controls. 4. substantive analytical procedures
d. Risk assessment. 5. tests of details of balances
a. 1, 2, and 3
31. In theory, which of the following would not have an b. 3, 4, and 5
effect on the extent of audit evidence examined by a c. 2, 3, and 5
CPA? d. 2, 3, and 4
a. The types of audit evidence gathered.
b. The type of opinion to be issued. 38. Which of the following is not one of the four decisions
c. The auditor's evaluation of internal control. about what evidence to gather and how much of it to
accumulate in the development of an audit program?

Page 3 of 7 www.teamprtc.com.ph AUD.1stPB5.23


TEAM PRTC

a. which audit procedures to use


43. The profit for the year ended Dec. 31, 2023 is misstated
b. which accounts must agree to the general ledger
by
c. when to perform the procedures
a. P 10,000 c. P350,000
d. what sample size to select for a given procedure
b. P190,000 d. P430,000
39. Audit evidence has two primary qualities for the auditor; 44. The working capital as of Dec. 31, 2023 is misstated by
relevance and reliability. Given the choices below, which a. P 10,000 c. P350,000
provides the auditor with the most reliable audit b. P190,000 d. P430,000
evidence?
a. general ledger account balances 45. Purchase cut-off procedures should be designed to
b. confirmation of accounts receivable balance a. Test whether all inventory owned by the entity is in
received from the client’s customer the possession of the entity at year-end.
c. internal memo explaining the issuance of a credit b. To detect whether merchandise included in the
memo physical count at year-end was not recorded as a
d. copy of month-end adjusting entries sale in the subsequent period.
c. Both a and b.
40. The Auditor's Responsibilities section of the standard d. Neither a nor b.
unmodified opinion audit report states that the audit is
designed to
a. discover all errors and/or irregularities. PROBLEM NO. 2
b. discover material errors and/or irregularities.
c. conform to generally accepted accounting You requested a depreciation schedule for Delivery Trucks
principles. of Apatheia Corp. showing the additions, retirements,
d. obtain reasonable assurance whether the depreciation and other data affecting the income of the
statements are free of material misstatement. entity in the 4-year period 2020 to 2023, inclusive. The
Delivery Trucks account consists of the following as of Jan.
PROBLEM NO. 1 1, 2020:

In connection with your audit of Stillness Corp. for the year Truck No. 1 purchased Jan. 1, 2017, cost P 180,000
ended Dec. 31, 2023, you found the following information Truck No. 2 purchased July 1, 2017, cost 220,000
relating to certain inventory transactions from your Truck No. 3 purchased Jan. 1, 2019, cost 300,000
observation of the client’s physical count and review of sales Truck No. 4 purchased July 1, 2019, cost 240,000
and purchases cutoff: P 940,000

a. Goods costing P180,000 were received from a vendor The Delivery Trucks–Accumulated Depreciation account
on Jan. 3, 2024. The related invoice was received and previously adjusted to Jan. 1, 2020, and duly entered to the
recorded on Dec. 30, 2023. The goods were shipped on ledger, had a balance on that date of P302,000
Dec. 31, 2023, terms FOB shipping point. (depreciation on the 4 trucks from respective date of
purchase, based on five-year life, no salvage value). No
b. Goods costing P200,000, sold for P300,000, were
charges have been made against the account before Jan. 1,
shipped on Dec. 31, 2023, and were received by the
2020.
customer on Jan. 2, 2024. The terms of the invoice were
FOB shipping point. The sale was recorded in 2024.
Transactions between Jan. 1, 2020 and Dec. 31, 2023, and
c. The invoice for goods costing P150,000 was received their record in the ledger were as follows:
and recorded as a purchase on Dec. 31, 2023. The
July 1, 2020 – Truck No. 3 was traded for larger one (No.
related goods, shipped FOB destination, were received
5), the agreed purchase price of which was
on Jan. 2, 2024.
P340,000. Cheerful Mfg. Co. paid the
d. A P600,000 shipment of goods to a customer on Dec. automobile dealer P150,000 cash on the
30, 2023, terms FOB destination, was recorded as a sale transaction. The entry was debit to Delivery
upon shipment. The goods, costing P400,000, were Trucks and a credit to Cash, P150,000.
received by the customer on Jan. 6, 2024.
Jan. 1, 2021 – Truck No. 1 was sold for P35,000 cash; entry
e. Goods costing P250,000 were received and recorded as debited Cash and credited Delivery Trucks,
a purchase on Dec. 31, 2023. These goods are held on P35,000.
consignment from a vendor.
July 1, 2022 – A new truck (No. 6) was acquired for
f. Goods costing P160,000, recorded as a sale upon P360,000 cash and was charged at that
shipment for P240,000, were shipped on Dec. 31, 2023. amount to Delivery Trucks account.
These goods are out on consignment with the customer (Assume truck No. 2 was not retired.)
and sold to a third party on Jan. 5, 2024.
July 1, 2022 – Truck No. 4 was damaged in a wreck to
such an extent that it was sold as junk for
QUESTIONS:
P7,000 cash. Cheerful Mfg. Co. received
Based on the given information and the result of your audit, P25,000 from the insurance company. The
answer the following: entry made by the bookkeeper was a debit
to cash, P32,000, and credits to
41. The inventory as of Dec. 31, 2023 is understated by
Miscellaneous Income, P7,000 and Delivery
a. P140,000 c. P490,000
Trucks P 25,000.
b. P330,000 d. P580,000
42. The cost of sales for the year ended Dec. 31, 2023 is Entries for depreciation had been made for the close of each
overstated by year as follows: 2020, P203,000; 2021, P211,000; 2022,
a. P290,000 c. P890,000 P244,500; 2023, P278,000.
b. P730,000 d. P980,000

Page 4 of 7 www.teamprtc.com.ph AUD.1stPB5.23


TEAM PRTC

QUESTIONS: QUESTIONS:
Based on the given information and the result of your audit, Based on the given information and the result of your audit,
determine the following: determine the following:
46. The 2020 profit is overstated by 51. Amortization of patent for 2023
a. P 9,000 c. P31,000 a. P400,000 c. P800,000
b. P20,000 d. P 0 b. P720,000 d. P900,000
47. The 2021 profit is understated by 52. Carrying amount of patent as of Dec. 31, 2023
a. P51,000 c. P16,000 a. P2,400,000 c. P2,880,000
b. P50,000 d. P 0 b. P2,700,000 d. P3,200,000
48. The 2022 profit is understated by 53. Carrying amount of intangible assets as of Dec. 31,
a. P23,500 c. P94,500 2023
b. P64,500 d. P 0 a. P3,264,000 c. P3,744,000
b. P3,564,000 d. P4,610,000
49. The adjusted carrying amount of Delivery Trucks as of
December 31, 2023 is 54. Total expenses to be recognized in 2023
a. P885,400 c. P354,000 a. P1,066,000 c. P2,012,000
b. P504,000 d. P284,000 b. P1,932,000 d. P2,112,000
50. Which statement is incorrect regarding audit of 55. Which statement is correct regarding audit of intangible
property, plant and equipment? assets?
a. Property, plant and equipment is typically judged to a. An auditor most likely would review or re-compute
be one of the accounts least susceptible to fraud amortization and determine whether the
because the inherent risk is usually low. amortization period is reasonable in support of
b. When few property and equipment transactions management’s financial statement assertion of
occur during the year the continuing auditor usually valuation.
obtains an understanding of internal control and b. The most effective means for the auditor to
performs extensive tests of current year property determine whether a recorded intangible asset
and equipment transactions. possesses the characteristics of an asset is to
c. Determining that proper amounts of depreciation inquire as to the status of patent applications.
are expensed provides assurance about c. When testing a client's additions to an asset for
management’s assertions of valuation and research and development, the auditor must
presentation and disclosure. remember that such costs should be amortized over
d. In testing for unrecorded retirements of equipment, the lesser of their legal lives or useful lives.
an auditor is most likely to inspect certain items of d. None of these.
equipment in the plant and trace those items to the
accounting records.
PROBLEM NO. 4
You were able to obtain the following information in
PROBLEM NO. 3
connection with your audit of the Cash account of the
In connection with your audit of the Upekkha Corporation’s Aslama Corp. as of Dec. 31, 2023:
financial statements for the year 2023, you noted the Nov. 30 Dec. 31
following items relative to the company’s Intangible Assets. a. Balances per bank P480,000 P420,000
b. Undeposited collections 244,000 300,000
• A patent was purchased for P4,000,000 on Jan. 2, 2022. c. Outstanding checks 150,000 120,000
Upekkha estimated that the remaining useful life of the
patent to be 10 years. The patent was carried in seller’s d. The bank statement for the month of Dec. showed total
accounting records at a carrying value of P4,000,000 credits of P240,000.
when sold to Upekkha.
e. DAIF checks are recorded as a reduction of cash
receipts. DAIF checks which are later redeposited are
• During 2023, a franchise was purchased for P960,000.
then recorded as regular receipts. Data regarding DAIF
In addition, 5% of the revenue from the franchise must
checks are as follows:
be paid to the franchisor. Revenue from the franchise
1. Returned by the bank in Nov. and recorded by the
for 2023 was P5,000,000. Upekkha estimates the useful
company in Dec., P10,000.
life of the franchise to be 10 years and takes full year’s
2. Returned by the bank in Dec. and recorded by the
amortization in the year of purchase.
company in Dec., P25,000.
3. Returned by the bank in Dec. and recorded by the
• Upekkha incurred research and development costs of
company in Jan., P29,000.
P866,000 in 2023. Upekkha estimates that these costs
will be recouped by 2025. f. Check of another company amounting to P90,000 was
charged to the Aslama’s account by the bank in error on
• On Jan. 1, 2023, Upekkha, because of the recent events Dec. 31.
in the industry, estimates that the remaining life of the
g. A bank memo stated that the company’s account was
patent purchased on Jan. 2, 2022, is only 5 years from
credited for the net proceeds of a customer’s note for
Jan. 1, 2023.
P106,000.
h. The company has hypothecated its accounts receivable
with the bank under an agreement whereby the bank
lends the company 80% of the hypothecated accounts
receivable. The company performs accounting and
collection of the accounts. Adjustments of the loan are
made from daily sales reports and deposits.

Page 5 of 7 www.teamprtc.com.ph AUD.1stPB5.23


TEAM PRTC

PROBLEM NO. 5
i. The bank credits the company account and increases the
amount of the loan for 80% of the reported sales. The In connection with the audit of the financial statements of
loan agreement states specifically that the sales report Hishtavut Corporation, your audit senior instructed you to
must be accepted by the bank before the company is examine the company’s accounts receivable.
credited. Sales reports are forwarded by the company
to the bank on the first day following the date of sale. Prior to any adjustments you were able to extract the
The bank allocates each deposit 80% to the payment of following balances from Hishtavut’s trial balance as of Dec.
the loan, and 20% to the company account. Thus, only 31, 2023:
80% of each day’s sales and 20% of each collection
Accounts receivable P442,500
deposits are entered on the bank statement. The
Allowance for doubtful accounts 15,000
company accountant records the hypothecation of new
accounts receivable (80% of sales) as a debit to Cash
From the schedule of accounts receivable as of Dec. 31,
and a credit to the bank loan as of the date of sales. One
2023, you determined that this account includes the
hundred percent of the collection on accounts receivable
following:
is recorded as a cash receipt; 80% of the collection is
recorded in the cash disbursements books as a payment Accounts with debit balances:
on the loan. In connection with the hypothecation, the 60 days old and below P238,500
following facts were determined: 61 to 90 days 117,200
• Included in the undeposited collections is cash from Over 90 days 85,400 P441,100
the hypothecation of accounts receivable. Sales Advances to officers 16,400
were P180,000 on Nov. 30, and P200,000 at Dec. Accounts with credit balance (15,000)
31. The balance was made up from collections which Accounts receivable per GL P442,500
were entered on the books in the manner indicated
above. The credit balance in customer’s account represents
• Collections on accounts receivable deposited in collection from a customer whose account had been written-
Dec., other than deposits in transit, totaled off as uncollectible in 2022.
P725,000.
Accounts receivable for more than a year totaling P21,000
j. Interest on the bank loan for the month of Dec. charged
should be written off.
by the bank but not recorded in the books, amounted to
P38,000.
Confirmation replies received directly from customers
disclosed the following exceptions:
QUESTIONS:
Based on the given information and the result of your audit, Customer’s
answer the following: Customer Comments Audit Findings
Ey The goods sold on The client failed to
56. How much is the unadjusted balance per books as of
Dec. 1 were record a credit memo
Nov. 30, 2023?
returned on Dec. for P12,000. The
a. P504,000 c. P430,000
16, 2023. merchandise was
b. P484,000 d. P356,000
included in the ending
57. How much is the unadjusted book receipts for Dec., inventory at cost.
2023?
a. P860,000 c. P735,000 Beh We do not owe this Investigation revealed
b. P770,000 d. P738,000 amount *%#@!!! that goods sold for
(censored). We did P16,000 were shipped
58. How much is the unadjusted book disbursements for
not receive any to Beh on Dec. 29,
Dec., 2023?
merchandise from 2023, terms FOB
a. P773,000 c. P735,000
your company. shipping point. The
b. P700,000 d. P760,000
goods were lost in
59. How much is the unadjusted balance per books as of transit and the
Dec. 31, 2023? shipping company has
a. P481,000 c. P309,000 acknowledged its
b. P530,000 d. P539,000 responsibility for the
loss of the
60. Which statement is incorrect regarding audit of cash? merchandise.
a. The general cash account is considered a significant
account in almost all audits even when the ending Sy I am entitled to a Sy is an employee of
balance is immaterial. 10% employee Hishtavut. Starting
b. The starting point for the verification of the balance discount. Your bill Nov. 2023, all
in the general bank account is to obtain a bank should be reduced company employees
reconciliation from the client. by P1,200. were entitled to a
c. An auditor who is engaged to examine the financial special discount.
statements of a business enterprise will request
cutoff bank statement primarily in order to verify
Deh We have not yet Merchandise billed for
the cash balance reported on the bank confirmation
sold the goods. P18,000 were
inquiry form.
We will remit the consigned to Deh on
d. Auditors are likely to prepare a proof of cash when
proceeds as soon Dec. 30, 2023. The
the client has material control weaknesses in cash
as the goods are goods cost P13,000.
receipts and cash disbursements.
sold.

Eh We do not owe you The sale of


P20,000. We merchandise on Dec.

Page 6 of 7 www.teamprtc.com.ph AUD.1stPB5.23


TEAM PRTC

Customer’s
Customer Comments Audit Findings PROBLEM NO. 6
already paid our 18, 2023 was paid by
Aequanimitas Corp. made investments equity and debt
accounts as Eh on Jan. 6, 2024.
instruments classified as FA at FVTPL. An analysis of these
evidenced by OR #
investments on Dec. 31, 2022 showed the following:
1234.
Instrument Cost Fair value
Ef Reduce your bill by This amount Marcus Inc. (6,000 shares) P307,500 P270,000
P1,500 represents freight
Cato Inc. (2,250 shares) 76,500 90,000
paid by the customer Seneca Co. 12% Bonds
for the merchandise (P300,000 face value) 269,500 280,600
shipped on Dec. 17, Total P653,500 P640,600
2023, terms, FOB
destination-collect. On Apr. 1, 2023, the entity purchased as a temporary
investment, P200,000 face value, 9% Philippine treasury
Based on your discussion with Hishtavut’s Credit Manager, notes for P198,500, which includes accrued interest. The
you both agreed that an allowance for doubtful accounts notes mature on July 1, 2024 and pay interest semiannually
should be maintained using the following rates: on Jan. 1 and July 1. The notes were sold on Dec. 1, 2023
60 days old and below 1% for P206,500, which includes accrued interest.
61 to 90 days 2%
Over 90 days 5% On July 1, 2023, the shares of Cato were sold for P70,000.
On Dec. 31, 2023, Marcus Inc. shares were quoted at P44
QUESTIONS: per share. Seneca bonds were quoted at P950 per P1,000
bond.
Based on the above and the result of your audit, answer the
following: QUESTIONS:
61. In relation to Customer Beh, the necessary adjusting Based on the above and the result of your audit, answer the
journal entry includes a debit to following:
a. Claim Against Shipping Company of P16,000
b. Sales of P16,000 66. The gain on sale of Philippine Treasury Notes on Dec. 1,
c. Loss of P16,000 2023 is
d. None of these a. P 500 c. P 8,000
b. P5,000 d. P12,500
62. In relation to Customer Deh, the necessary adjusting
journal entry includes 67. The loss on sale of Cato shares on July 1, 2023 is
a. A debit to Inventory of P18,000. a. P 0 c. P18,650
b. A credit to Cost of Sales of P13,000 b. P6,500 d. P20,000
c. A credit to Sales of P18,000 68. The interest income for the year 2023 is
d. None of these a. P24,000 c. P37,500
63. The adjusted balance of accounts receivable as of Dec. b. P36,000 d. P48,000
31, 2023 is 69. The carrying amount of the investments at Dec. 31,
a. P371,400 c. P402,400 2023 is
b. P387,400 d. P403,800 a. P520,025 c. P549,000
64. The adjusted allowance for doubtful accounts as of Dec. b. P533,500 d. P577,000
31, 2023 is 70. The net unrealized loss to be recognized in 2023 profit
a. P7,462 c. P7,786 or loss is
b. P7,622 d. P8,372 a. P 0 c. P 2,800
65. Which statement is incorrect regarding audit of b. P1,600 d. P15,100
receivables?
a. Unreturned negative confirmation requests rarely
provide significant explicit evidence. Thank you for participating in Team PRTC
b. An auditor should perform alternative procedures to Nationwide Online Open First Pre-Board
substantiate the existence of accounts receivable Examination.
when no reply to a negative confirmation request is
received.
c. When designing audit procedures, tracing of source
documents to the customers subsidiary ledger and
subsequently to the general ledger is done to verify
completeness assertion.
d. An auditor most likely would analyze notes
receivable and interest income on the same working
paper.

Page 7 of 7 www.teamprtc.com.ph AUD.1stPB5.23


Excel Professional Services Inc.
Management Firm of Professional Review and Training Center (PRTC)
Online • Manila • Cavite • Laguna • Cebu • Cagayan De Oro • Davao
Since 1977

Taxation (TAX) GUDANI/NARANJO/SIAPIAN/WONG


FIRST PRE-BOARD EXAMINATION CPA Review February 19, 20 & 21, 2023

Multiple Choice. Select the letter that corresponds to the best answer. This examination consists of 70 items and the
exam is good for three (3) hours. Good luck!

Use the tax tables if necessary:


REVISED WITHHOLDING TAX TABLE FOR COMPENSATION
DAILY 1 2 3 4 5 6
Compensation 685 and 685-1,095 1,096-2,191 2,192-5,478 5,479-21,917 21,918 and
Range below above
Prescribed 0 0 82.19 356.16 1,342.47 6,602.74
Withholding Tax + 20% over + 25% over + 30% over + 32% over + 35% over
685 1,096 2,192 5,479 21,918
WEEKLY 1 2 3 4 5 6
Compensation 4,808 and 4,808-7691 7,692-15,384 15,385-38,461 38-462- 153,846 and
Range below 153,845 above
Prescribed 0 0 576.92 2,500 9,423.08 46,346.15
Withholding Tax + 20% over + 25% over +30% over + 32% over + 35% over
4,808 7,692 15,385 38,462 153,846
SEMI-MONTHLY 1 2 3 4 5 6
Compensation 10,417 and 10,417- 16,667- 33,333-83,332 83,333- 333,333 and
Range below 16,666 33,332 333,332 above
Prescribed 0 0 1,250 5,416.67 20,416.67 100,416.67
Withholding Tax + 20% over + 25% over + 30% over + 32% over + 35% over
10,417 16,667 33,333 83,333 333,333
MONTHLY 1 2 3 4 5 6
Compensation 20,833 and 20,833- 33,333- 66,667- 166,667- 666,667 and
Range below 33,332 66,666 166,666 666,666 above
Prescribed 0 0 2,500 10,833.33 40,833.33 200,833.33
Withholding Tax + 20% over + 25% over + 30% over + 32% over + 35% over
20,833 33,333 66,667 166,667 666,667

SCHEDULE OF EXPANDED WITHHOLDING TAX (EWT)


Income Payment EWT Income Payment EWT
Professional/Management/Consultancy Gross receipts 3M or Purchase of goods or properties 1% - goods
fees (Individual) below - 5% by Top Withholding Agents 2% - service
Gross receipts over (TWA) -
3M or VAT-reg –
10%
Professional/Management/Consultancy Gross receipts Partners in general professional 720,000 or less
fees (Non-Individual) 720,000 or below - partnerships (drawings, -10%
10% advances, sharings, allowances, Above 720,000
Gross receipts over etc.) – 15%
720,000 – 15%
Contractors/subcontractors (security, 2% Commission (if not employee) 10%
janitorial, etc.)
Director’s fees (if not employee) 10% or 15% Income distributed to 15%
(720,000 threshold) beneficiaries of estates/trusts
Rental 5%

GRADUATED TAX TABLE UNDER TRAIN LAW (January 1, 2018 to December 31, 2022)
Over But not over The tax shall be Plus Of excess over
250,000 0 0 -
250,000 400,000 0 20% 250,000
400,000 800,000 30,000 25% 400,000
800,000 2,000,000 130,000 30% 800,000
2,000,000 8,000,000 490,000 32% 2,000,000
8,000,000 2,410,000 35% 8,000,000

GRADUATED TAX TABLE UNDER TRAIN LAW (January 1, 2023 and onwards)
Over But not over The tax shall be Plus Of excess over
0 250,000 0 0 -
250,000 400,000 0 15% 250,000
400,000 800,000 22,500 20% 400,000
800,000 2,000,000 102,500 25% 800,000
2,000,000 8,000,000 402,500 30% 2,000,000
8,000,000 - 2,202,500 35% 8,000,000

Page 1 of 7 www.teamprtc.com.ph TAX.1stPB5.23


1. S1 – Assessment of tax liabilities may be made by the b. 1601-EQ– 2nd Quarter July 31
BIR even in the absence of levy provided it can be Expanded
proven that the taxpayer earned an income. Withholding Tax
S2 – Collection can only be made if an assessment has c. 1601-FQ – Final 4th Quarter January
been issued in accordance with the imposition made by Tax 31
the Congress of the Philippines. d. 1603 – Fringe 3rd Quarter October
a. Only S1 is true c. Both are true Benefits Tax 31
b. Only S2 is true d. Both are false
9. S1 - Capital asset can be converted to ordinary asset if
2. The BIR raided warehouses where excisable articles with the same remained idle for more than two (2) years.
unpaid excise tax are kept. This power is based on S2 – Real properties owned by a real estate developer
BIR’s: cannot be converted to capital asset even if the assets
a. Police Power are abandoned after two (2) years.
b. Power of Eminent Domain Which is TRUE?
c. Power of Taxation a. Only S1 is true c. Both are true
d. Power of Unity b. Only S2 is true d. Both are false

3. Which is FALSE? 10. Which is TRUE? The Commissioner of Internal Revenue


I. If an individual taxpayer opted for 8% taxation, S1 -Can close the business without due process
he is disqualified from using OSD. provided that there is sufficient proof that taxpayer
II. If an individual taxpayer opted OSD as his evaded payment of tax.
method of deduction, he is disqualified from S2 - Can arrest taxpayers who, in his presence, violated
availing 8% preferential tax. the provisions of the Tax Code.
a. Only S1 is true c. Both are true S3 - Can seize excisable articles with unpaid excise
b. Only S2 is true d. Both are false taxes.
a. S1 only c. S3 only
4. Which is TRUE? De minimis benefits are: b. S2 only d. All are true
S1 – Exempt from income tax but subject to withholding
tax. 11. Which is TRUE? Assume all incomes are derived from
S2 – Subject to fringe benefits tax or withholding tax on within the Philippines.
compensation if the threshold is exceeded. I. Nicanor, a resident citizen who is an employee
a. Only S1 is true c. Both are true has no other option but to use graduated tax
b. Only S2 is true d. Both are false rate on his compensation.
II. Nick Aynor, a Filipino contract worker in the USA
5. Which is taxable? has the option to use 8% taxation on his
S1 – Php 5,000.00 excess of the allowable uniform business income derived from sources within
allowance where the employee’s actual 13th month pay the Philippines.
and other benefits amounted to Php 50,000.00. III. Nica Knorr, a Filipino contract worker in the USA
S2 - Daily tricycle allowance of Php 50.00 to a rank-and- can use graduated tax rates in computing his
file employee. income.
S3 - Reimbursable monthly representation expense IV. Nick Nore, a Filipino contract worker in the USA
given to managerial employee. and with business in the Philippines is a mixed
S4 – Productivity incentive of Php 5,000.00. income earner.
a. S1 and S2 only c. S1, S2 and S3 only a. I and II only c. I, II and IV only
b. S2 and S3 only d. All of the above b. I and IV only d. All are true

6. S1 - The taxpayer can temporarily use the withholding 12. S1 – On her 21st birthday, Nicanor was gifted a new car
taxes for working capital requirements. by his girlfriend Inday. The income of Nicanor (car) is a
S2 – If income earner reported the income, the exempt from income taxation.
withholding agent is exonerated from his withholding S2 – Nicanor, a senior citizen, resigned from ABC Corp.
tax obligation. where he was working for the last thirty years. His
a. Only S1 is true c. Both are true separation pay is exempt from taxation.
b. Only S2 is true d. Both are false S3 – Nicanor, an employee, sold his house and lot for
Php 5 Million. He bought the said property five years ago
7. The following persons are exempt from withholding tax, for Php 1 Million. Nicanor is required to report the
except: income.
S1 - Government instrumentalities. S4 – Nicanor, a corrupt government employee, receives
S2 - Fiscal Incentives Review Board/Investment “gifts” in exchange for his processing of applications of
Promotion Agency-registered enterprises with Income taxpayers. The money he receives is exempt from
Tax Holidays. taxation.
S3 – Ali and Baba, Attorneys-at-Law, a general Which is FALSE?
professional partnership. a. S1 c. S3
S4 – Philippine Human Eagles, a non-stock, non-profit b. S2 d. S4
organization. e. None
a. S2 only
b. S2 and S4 only 13. The BIR received a confidential information that ABC
c. S1, S2 and S3 only Corp. was evading taxes. A notice of audit was served
d. All are exempt from withholding to the Company. The Company refused to provide the
necessary documents in relation to the audit. To compel
8. Which is FALSE as to the deadline? the taxpayer to produce the documents, the
Return Period Deadline Commissioner or his authorized representative may use
a. 1601-C – December January his:
Withholding on 10 a. Power to examine
Compensation b. Power to obtain information

Page 2 of 7 www.teamprtc.com.ph TAX.1stPB5.23


TEAM PRTC

c. Power to summon d. 25% on the portion of the revenue of a non-stock,


d. Power to terminate taxable period non-profit educational institution not used directly
and exclusively for educational purpose.
14. The BIR received a confidential information that ABC
Corp. was evading taxes. To examine tax returns and 21. Which is TRUE? In ordinary partnership is:
accounting records and ensure validity of the S1 - Distribution of income to partners are deemed
assessment, the BIR must issue: dividends generally subject to 10% final tax.
a. A Certificate Authorizing Registration S2 - Share in net income are required to be reported by
b. A Mission Order the partners in their individual and separate capacity.
S3 - The partnership is subject to regular corporate
c. An authorization letter income tax of 25% or 20%.
d. An electronic Letter of Authority a. S1 only c. S2 and S3 only
15. The BIR received a confidential information that ABC b. S2 only d. S1 and S3 only
Corp. was evading taxes. To determine under-
declaration of income, the BIR may conduct inventory- 22. In the deadline of filing of 2021 Annual Income Tax
taking or surveillance. The document required to be Returns, which is TRUE?
issued by the BIR is: S1 – ABC Corp. employing fiscal year ending January
31, the deadline is May 15 of the same year.
a. A Certificate Authorizing Registration S2 – ABC Corp. employing fiscal year ending June 30,
b. A Mission Order the deadline is October 15 of the same year.
c. An authorization letter S3 – ABC Corp. employing fiscal year ending October
d. An electronic Letter of Authority 31, the deadline is February 15, 2022.
S4 – ABC Corp. employing calendar, the deadline is April
16. Which is FALSE in withholding tax system? 15, 2022.
S1 – Only certain passive incomes derived from sources a. S1 and S2 c. S1, S2 and S4
within the Philippines are subject to final withholding b. S2 and S3 d. All are true
tax.
S2 – All income payments are subject to withholding 23. Which is TRUE in fringe benefits tax?
tax. S1 – Monetary value of fringe benefit less fringe benefits
S3 – If an income is exempted from taxation, it is also is equal to fringe benefits tax.
exempt from withholding. S2 – Grossed-up monetary value of fringe benefits less
S4 – The income earner in withholding tax system is of monetary value of fringe benefits is equal to fringe
required to report the income subjected to any benefits tax.
withholding tax. S3 – Fringe benefits tax plus monetary value of fringe
a. S2 and S3 c. S2, S3 and S4 benefits is equal to grossed-up monetary value of the
b. S2 and S4 d. All are false fringe benefits.
a. S1 and S2 c. S2 and S3 only
17. Nicanor bough preferred shares at par value from ABC b. S1 and S3 d. All are true
Corp. If he will sell, the preferred shares, the fair market
value shall be computed using: 24. Who is NOT required to file Annual Income Tax Return
a. Book value c. Par value (AITR)?
b. Liquidation value d. Selling price No. of Income Tax Tax
Employer Due Withheld
18. Sale of real property is subject to: by
S1 – capital gains tax if property is classified as a capital Employer
asset. Nicanor 1 Php 10,000 Php 10,000
S2 – creditable withholding tax if property is classified Jose 2 (minimum Php 0 Php 0
as ordinary asset. wage
Which is TRUE? earner)
a. Only S1 c. Both are true Andres 1 Php 10,000 Php 11,000
b. Only S2 d. Both are false Goyo 2 Php 10,000 Php 10,000

19. Which is TRUE? ABC College, an educational institution: a. Nicanor only


S1 – is exempt from income tax if it is a non-stock, non- b. Nicanor and Goyo
profit educational institution.
c. Nicanor, Jose and Andres
S2 - is subject to regular tax if it operates without
accreditation or permit from DepEd, CHEd or TESDA. d. All are not required to file
S3 – is subject to preferential tax if with permit from
DepEd, CHEd or TESDA and income from related activity 25. Which is TRUE?
is higher than unrelated activity. a. Hospitals for profit are subject to preferential tax of
a. S1 and S2 c. S2 and S3 10%
b. S1 and S3 d. All are true b. Offshore banking units are subject to 10%
preferential tax
20. Which is FALSE? A proprietary education institution is c. Regional area headquarters are subject to 10%
subject to: preferential tax
a. 25% regular rate if income from unrelated activity d. Regional operating headquarters are subject to 25%
is higher than related activity. regular tax
b. 10% rate if it is a non-stock corporation with duly
issued permit from DepEd, CHEd or TESDA. 26. S1 - The MCIT is imposed beginning on the fourth
taxable year immediately following the year in which
c. 10% if taxpayer is an individual.
such corporation commenced its business operations

Page 3 of 7 www.teamprtc.com.ph TAX.1stPB5.23


TEAM PRTC

S2 - When the minimum corporate income tax is higher


than the regular corporate income tax during the period, 33. Nicanor owns a sports utility vehicle which he plans to
the minimum corporate income tax shall be payable sell. As a friend, he asked you when the best time is to
S3 - Excess of minimum corporate income tax can be sell his car. You advise him to sell it after twelve (12)
carried over and credited to the regular income tax for months after the date he acquires it because under the
the next three immediately succeeding taxable years law, the capital gain to be recognized is 50% only. This
S4 - MCIT applies on quarterly Income Tax Returns is an example of:
Which is TRUE? a. Tax avoidance c. Tax exemption
a. S1 and S3 c. S1, S2 and S3 only b. Tax evasion d. Tax Shifting
b. S1 and S2 only d. All are true
34. Joshua orally donated a land worth P 3,000,000 to
27. S1: For income tax purposes, domestic corporations Bulet.
shall account separately in their Annual Financial S1: The land received by Bulet is an income.
Statements (AFS) the cost of the land on which the S2: The land received by Bulet is subject to Income tax.
particular business entity’s office, plant and equipment a. Only S1 is true c. Both are true
are situated, and shall not lump the same in one account b. Only S2 is true d. Both are false
title nor consolidate its cost with fixed asset account.
S2: GOCC’s, agencies and instrumentalities, except the 35. Michael sold his land worth P 10,000,000 to Georgia.
GSIS, the SSS, the HDMIF, the PHIC, and the local water S1: Assuming Michael is engaged in the business of real
districts, shall pay such rate of tax upon their taxable estate, the sale of land is subject to capital gains tax.
income as are upon corporations or association engaged S2: Assuming Michael is not engaged in the business of
in a similar business, industry, or activity. real estate but the land he sold is used in his business,
a. Only S1 is true c. Both are true the sale of land is subject to capital gains tax.
b. Only S2 is true d. Both are false S3: Assuming Michael is not engaged in the business of
real estate, the sale of land is subject to capital gains
28. S1: Income derived by an international air carrier is tax.
always subject to 2.5% Gross Philippine Billings tax. a. Only S1 is true c. Only S3 is true
S2: International air carrier may be exempted from b. Only S2 is true d. All is false
income tax.
a. Only S1 is true c. Both are true 36. In 2023, Nicanor is employed by Life Insurance Corp. as
b. Only S2 is true d. Both are false an accountant. His taxable compensation for the year
amounts to Php 1,200,000 inclusive of Php100,000, 13th
29. On August 15, 2022, ABC Corporation celebrated its month pay. During the same year, he was able to earn
50th anniversary. It grants stock options to its ten (10) Php 2,000,000 as a commission from Moon Insurance
rank-and-file employees who have stayed with the Corp (another insurance company). In filing his Income
company from the date of its incorporation as well as its Tax Return for the calendar year 2023, he opted to
five (5) sales managers, who have contributed a lot to choose 8% income tax. Nicanor’s proper income tax
the company. The following year in March 2023, all of due to pay is?
the said employees availed the said option. a. Php 362,500.00 c. Php 320,000.00
S1: In 2023, the stock options exercised by the ten (10) b. Php 337,500.00 d. Php 340,000.00
rank-and-file employees are subject to withholding tax
on compensation. 37. In 2023, Inday sold her rest house in Bicol for Php
S2: In 2023, the stock options exercised by the five (5) 8,000,000. The fair market value of the rest house at
sales managers are subject to fringe benefits tax. the time of sale is Php 10,000,000. Inday utilized Php
a. Only S1 is true c. Both are true 6,000,000 of the proceeds of the sale in acquiring a new
principal residence, the capital gains tax due is?
b. Only S2 is true d. Both are false
a. P 600,000 c. P 480,000
b. P 360,000 d. P 0
30. S1: The grossed-up monetary value of the fringe
benefit is the actual amount received by the employee.
Use the following information to answer next four (4)
S2: Premiums paid by the employer for the group
questions.
insurance of its managers are subject to fringe benefit
tax.
Nicanor made available the following financial information
a. Only S1 is true c. Both are true for the year 2023:
b. Only S2 is true d. Both are false
Gross receipts - Php 5,500,000
31. S1: Sale of real property classified as capital asset to Cost of service – 2,000,000
the government is subject to 6% capital gains tax or Expenses:
ordinary income tax, at the option of any taxpayer. Salaries and wages – Php 800,000
S2: Sale of real property classified as ordinary asset to
Rental 285,000
the government is subject to 6% capital gains tax or
Fines and Penalties 85,000 Note 1
ordinary income tax, at the option of any taxpayer.
Representation expense 75,000
a. Only S1 is true c. Both are true
Transportation & travel 25,000
b. Only S2 is true d. Both are false
Depreciation expense 15,000
32. All of the following, except one, is an essential Supplies expense 10,000
characteristic of a tax: Miscellaneous expenses 10,000
S1: It is based in the ability to pay.
S2: It is an enforced contribution. BIR Form 2307 (EWT for 1st 275,000
S3: It is generally payable in money. Quarter to 4th Quarter)
S4: It is proportionate in character. Tax payments for the first 180,620
a. Only S1 c. Only S3 Three (3) Quarters
b. Only S2 d. Only S4

Page 4 of 7 www.teamprtc.com.ph TAX.1stPB5.23


TEAM PRTC

Note 1
Compromise P 55,000 44. Based on the above information, how much is exempted
Surcharges 28,000 from income tax, if any?
Interest 2,000 a. P 72,000 c. P 80,000
Fines and Penalties (due to late filing of tax b. P 0 d. P 92,000
return) 85,000
45. From the facts, how much is the final tax, if any?
38. How much is the total allowable deduction assuming a. P 9,300 c. P 0
Nicanor opted itemized deduction as his method of b. P 10,200 d. 11,000
deduction?
a. Php 1,305,000 c. Php 1,220,000 Use the following information to answer next six (6)
b. Php 1,202,000 d. Php 1,277,000 questions.

39. In relation to the above question, how much is the In 2022, ABC University, a Commission on Higher
income tax payable of Nicanor? Education (CHEd)-accredited educational institution,
a. Php 129,740 c. Php 921,880 shared the following information:
b. Php 36,280 d. Php 105,740
Revenues:
40. Assuming Nicanor opted 8% Income tax regime, how Basic tuition fees 20,000,000.00
much is the correct income tax payable of Nicanor? Library and computer fees 1,000,000.00
a. Php 129,740 c. Php 921,880 Graduation fees 1,000,000.00
b. Php 36,280 d. Php 105,740 Certification fees 1,000,000.00
NSTP Training fees 1,000,000.00
41. Assuming Nicanor opted Optional Standard Deduction, Miscellaneous school fees 2,000,000.00
how much is the income tax payable/(Overpayment) of Sale of books 1,500,000.00
Nicanor? Sale of uniforms 2,500,000.00
a. Php 450,380 c. Php (58,120) Rental income (from concessionaires) 10,000,000.00
b. Php 336,880 d. Php 423,180 Total 40,000,000.00

42. On May 12, 2023, Inday’s house and lot in Makati was Cost of service/sales:
totally destroyed by fire. The property had an adjusted Cost of service (school-related) 10,000,000.00
basis and a fair market value of Php 3,000,000 before Cost of books 1,000,000.00
the fire. During the same taxable year, Inday received
Cost of uniforms 1,000,000.00
Php 2,500,000 as reimbursement from Fire Insurance
Cost of rental 3,000,000.00
Company for the destruction of her home. What amount
Total 15,000,000.00
of fire loss was Inday entitled to claim as an itemized
deduction in 2023 income tax return assuming she has
P 10,000,000 Gross income from business? Expenses:
a. P 0 c. P 3,000,000 Salaries and wages (admin) 5,000,000.00
b. P 500,000 d. P 2,500,000 Gas and oil 1,000,000.00
Depreciation 2,000,000.00
Use the following information to answer next three (3) Utilities expenses 3,000,000.00
questions. Professional fees 3,000,000.00
Miscellaneous expenses 2,000,000.00
Nicanor retired at the end of 2022 after rendering 25 years Expenses attributable to rental 2,000,000.00
of continuous services for ABC Corporation which employed Total 18,000,000.00
him immediately after his graduation from college. Nicanor
was then 23 years old when he graduated from college. The Other information:
corporation, for the past 25 years, does not have a Assets:
Collective Bargaining Agreement with its employees and no Cash 20,000,000.00
labor organization was ever formed. He earned the following Accounts Receivable 10,000,000.00
income in 2022: Office Equipment 10,000,000.00
Building 50,000,000.00
Salary for the first quarter 180,000 Liabilities 30,000,000.00
Honorarium as speaker in one of ABC 10,000 Equity/Capital 60,000,000.00
corporation’s team building activities
Retirement Pay 2,500,000 46. The income tax due of ABC University assuming it is a
Commissions 30,000 domestic stock corporation is:
Fee as a member of ABC Corporation board 50,000 a. Php 70,000.00 c. Php 1,400,000.00
of directors
b. Php 700,000.00 d. Php 1,750,000.00
10 days monetized vacation leave 12,000
Interest income from time deposit in 24,000 47. The income tax due of ABC University assuming it is a
BDO-Lipa (net of final withholding tax) domestic non-stock corporation is:
Dividend income from XYZ Corporation (as 45,000
a. Php 70,000.00 c. Php 1,400,000.00
an investor, net of final withholding tax)
Productivity incentive pay 20,000
b. Php 700,000.00 d. Php 1,750,000.00
13th Month pay 60,000
48. The income tax due of ABC University assuming it is a
non-stock, non-profit institution and with its income
43. Based on the above information, how much is the Gross
being directly and exclusively used for education
Compensation Income of Nicanor?
purposes:
a. P 2,710,000 c. P 270,000
b. P 2,680,000 d. P 2,770,000
a. Php 0 c. Php 700,000.00
b. Php 70,000.00 d. Php 1,750,000.00

Page 5 of 7 www.teamprtc.com.ph TAX.1stPB5.23


TEAM PRTC

49. The income tax due of ABC University assuming it is a 53. The income tax due of ABC University assuming it is a
non-stock, non-profit institution and with its income domestic non-stock corporation is:
NOT directly and exclusively used for education a. Php 0 c. Php 300,000.00
purposes and inures to the benefits of the members: b. Php 30,000.00 d. Php 750,000.00
a. Php 0 c. Php 700,000.00
b. Php 70,000.00 d. Php 1,750,000.00 54. The income tax due of ABC University assuming it is a
non-stock, non-profit institution and with its income
50. The minimum corporate income tax of ABC University being directly and exclusively used for education
assuming it is domestic stock proprietary institution is: purposes:
a. Php 0 c. Php 180,000.00 a. Php 0 c. Php 300,000.00
b. Php 70,000.00 d. Php 250,000.00 b. Php 30,000.00 d. Php 750,000.00

51. The income tax due of ABC University assuming it has 55. The income tax due of ABC University assuming it is a
no accreditation as an educational institution: non-stock, non-profit institution and with its income
a. Php 0 c. Php 700,000.00 NOT directly and exclusively used for education
b. Php 70,000.00 d. Php 1,750,000.00 purposes and inures to the benefits of the members:
a. Php 0 c. Php 300,000.00
Use the following information to answer next five (5) b. Php 30,000.00 d. Php 750,000.00
questions.
56. The income tax due of ABC University assuming it has
Consider the information answering the following no accreditation as an educational institution:
questions: a. Php 0 c. Php 300,000.00
b. Php 30,000.00 d. Php 750,000.00
In 2022, ABC University shared the following information:
57. ABC Corp., a resident foreign corporation with an asset
Revenues: of Php 50,000,000.00, showed the following
Basic tuition fees 20,000,000.00 information on his taxable year 2022 operations:
Library and computer fees 1,000,000.00
Graduation fees 1,000,000.00 Gross Receipts 20,000,000.00
Certification fees 1,000,000.00 Less: Cost of Service 10,000,000.00
NSTP Training fees 1,000,000.00 Gross income 10,000,000.00
Miscellaneous school fees 2,000,000.00 Less: Allowable Deductions 8,000,000.00
Sale of books 1,500,000.00 Net Income 2,000,000.00
Sale of uniforms 2,500,000.00
Rental income (from concessionaires) 20,000,000.00 The income tax due is:
Total 50,000,000.00 a. Php 200,000.00 c. Php 500,000.00
b. Php 400,000.00 d. Php 600,000.00
Cost of service/sales:
Cost of service (school-related) 10,000,000.00 58. ABC Corp., a regional operating headquarters of ABC
Cost of books 1,000,000.00 Global Group based in Canada showed the following
Cost of uniforms 1,000,000.00 information on his taxable year 2022 operations:
Cost of rental 1,000,000.00
Total 13,000,000.00 Gross Receipts 20,000,000.00
Less: Cost of Service 10,000,000.00
Expenses: Gross income 10,000,000.00
Salaries and wages (admin) 5,000,000.00 Less: Allowable 8,000,000.00
Gas and oil 1,000,000.00 Deductions
Depreciation 2,000,000.00 Net Income 2,000,000.00
Utilities expenses 3,000,000.00 a. Php 0 c. Php 500,000.00
Professional fees 3,000,000.00 b. Php 400,000.00 d. Php 600,000.00
Miscellaneous expenses 2,000,000.00
Expenses attributable to rental 18,000,000.00 Use the following information to answer next seven (7)
Total 34,000,000.00 questions.

Other information: Consider the information answering the following


Assets: questions:
Cash 20,000,000.00
Accounts Receivable 10,000,000.00 A taxpayer’s financial records for the taxable year 2022
Office Equipment 10,000,000.00 are as follows:
Building 50,000,000.00
Liabilities 30,000,000.00 Source: Philippines Canada*
Equity/Capital 60,000,000.00 (translated
to peso)
52. Assuming ABC University is an accredited educational Gross income 5,000,000 10,000,000
institution, the income tax due is: Allowable deductions 3,000,000 5,000,000
a. Php 0 c. Php 300,000.00 Interest income from
b. Php 30,000.00 d. Php 750,000.00 lending 1,000,000 1,000,000
Interest income from bank
deposits 1,000,000 1,000,000

Page 6 of 7 www.teamprtc.com.ph TAX.1stPB5.23


TEAM PRTC

Dividends 1,000,000 1,000,000


Passive Royalty 1,000,000 1,000,000
67. How much is the tax savings if Nicanor opted for
59. Assuming the taxpayer is a resident citizen, how much optional standard deduction as his method of
is the taxable income subject to regular tax? deduction instead of itemized deduction?
a. Php 12,000,000 c. Php 14,000,000 a. Php 0 c. Php 560,000
b. Php 13,000,000 d. Php 15,000,000 b. Php 290,000 d. Php 1,280,000

60. Assuming the taxpayer is a resident citizen, how much Use the following information to answer next two (2)
is the passive income derived from sources in the questions.
Philippines which is subject to final tax?
The following cash benefits were given to Nicanor, a
a. Php 3,000,000 c. Php 6,000,000
government employee during December 2022:
b. Php 4,000,000 d. Php 8,000,000 • Monetized vacation leave credits (20 days at Php 1,000
per day) – Php 20,000
61. Assuming the taxpayer is a resident citizen, how much • Monetized sick leave credits (20 days at Php 1,000 per
is the total final tax withheld/paid? day) – Php 20,000
a. Php 500,000 c. Php 800,000 • Uniform allowance – Php 10,000
b. Php 700,000 d. Php 1,000,000 • Christmas cash gift – Php 10,000
• Employee achievement award (loyalty award) – Php
62. Assuming the taxpayer is a domestic corporation, how 10,000
much is the taxable income subject to regular tax? • Annual medical assistance – Php 10,000
a. Php 12,000,000 c. Php 14,000,000
b. Php 13,000,000 d. Php 15,000,000 68. You were preparing the alpha list of employees, how
much is the total de minimis benefits that should
63. Assuming the taxpayer is a domestic corporation, how reported?
much is the income exempt from taxation? a. Php 61,000 c. Php 71,000
a. Php 0 c. Php 2,000,000 b. Php 66,000 d. Php 76,000
b. Php 1,000,000 d. Php 3,000,000
69. You were preparing the Certificate of Compensation
Payment / Tax Withheld (BIR Form 2316) of Nicanor.
64. Assuming the taxpayer is a resident foreign
corporation, how much is the income exempt from How much is the total excess de minimis benefits that
taxation? should form part of the Php 90,000 threshold?
a. Php 4,000 c. Php 19,000
a. Php 10,000,000 c. Php 12,000,000
b. Php 9,000 d. Php 29,000
b. Php 11,000,000 d. Php 13,000,000
70. Nicanor sold his real property located in Apalit,
65. Assuming the taxpayer is a non-resident foreign Pampanga. The details of information were duly
corporation, how much is the income exempt from provided:
gross income/final tax? • Consideration in the Deed of Absolute Sale – Php
a. Php 9,000,000 c. Php 11,000,000 5,000,000
b. Php 10,000,000 d. Php 12,000,000 • Zonal value – Php 4,000,000
• FMV of assessor – Php 3,000,000
66. Nicanor is a supplier of goods. He was tagged by the • Mortgage value of land as annotated in the back of
BIR as a Top Withholding Agent. The 2022 records the Certificate of Title – Php 7,000,000
from his books of accounts disclosed the following: • Prevailing selling price in the area – Php 6,000,000
• Imputed selling price of BIR Revenue Officer due to
Gross Sales 10,000,000.00 commercialization of area – Php 10,000,000
Cost of Sales: What is the basis in computing the capital gains tax?
Beginning Inventory 2,000,000.00 a. Php 5,000,000 c. Php 7,000,000
Add: Purchases 4,000,000.00 b. Php 6,000,000 d. Php 10,000,000
Goods Available for Sale 6,000,000.00
Less: Ending Inventory 1,000,000.00 Thank you for participating in Team PRTC
Cost of sales 5,000,000.00 Nationwide Online Open First Pre-Board
Gross Profit 5,000,000.00 Examination.
Less: Expenses:
Salaries and wages 500,000.00
Rent expense 500,000.00
Gas and oil 500,000.00
Depreciation 500,000.00
Utilities expenses 500,000.00
Professional fees 500,000.00
Net income 2,000,000.00

How much is the tax savings assuming he will opt for


graduated tax instead of 8% preferential tax?
a. Php 0 c. Php 310,000
b. Php 290,000 d. Php 490,000

Page 7 of 7 www.teamprtc.com.ph TAX.1stPB5.23


Excel Professional Services Inc.
Management Firm of Professional Review and Training Center (PRTC)
Online • Manila • Cavite • Laguna • Cebu • Cagayan De Oro • Davao
Since 1977

Regulatory Framework for


Business Transactions (RFBT)
FIRST PRE-BOARD EXAMINATION
CPA Review VILLEGAS/APRADO/MAGUMUN
February 19, 20 & 21, 2023

Multiple Choice. Select the letter that corresponds to the


best answer. This examination consists of 100 items and 6. Lemon Law Rights period is:
the exam is good for three (3) hours. Good luck! a. within ten (10) months from the date of original
delivery to the consumer, or up to fifteen thousand
1. The following are the instances when an unpaid seller (15,000) kilometers of operation after such
can exercise the right to resell. Which one should not be delivery, whichever comes first.
in the list? b. within twelve (10) months from the date of original
a. When he has either a right of lien or a right to stop delivery to the consumer, or up to eighteen
the goods in transitu and the goods are perishable thousand (18,000) kilometers of operation after
in nature such delivery, whichever comes first.
b. When he has either a right of lien or a right to stop c. within twelve months from the date of original
the goods in transitu and the buyer delays in the delivery to the consumer, or up to twenty thousand
payment of the price for an unreasonable time (20,000) kilometers of operation after such sale,
c. When he has either a right of lien or a right to stop whichever comes first.
the goods in transitu and the right to resell is d. within twelve months from the date of original
expressly reserved in case the buyer should make a delivery to the consumer, or up to twenty thousand
default (20,000) kilometers of operation after such
delivery, whichever comes first.
d. When he has either a right of lien or a right to stop
the goods in transitu and the buyer has sold the
7. Which of the following is not a remedy of a vendor in
goods to another person
sale of personal property payable in instalments?
2. In case of violation of the EDBA, to which government a. Exact fulfillment of the obligation, should the
agency can the aggrieved party file a complaint? vendee fail to pay
i. Regional Trial Court where the agency b. Cancel the sale, if the vendee shall have failed to
concerned is located pay two or more installments
ii. Civil Service Commission c. Foreclose the chattel mortgage, if the vendee shall
iii. Office of the Ombudsman have failed to pay two or more installments
iv. Department of Justice d. Cancel the sale, if the vendee shall have failed to
a. i, ii, and iii only pay one installment
b. i, ii, and iv only
c. ii and iii only 8. If a bid price, as evaluated and calculated, is higher than
the Approved Budget of the Contract, what shall the BAC
d. iii and iv only do?
a. Disqualify the bidder automatically
3. In case of incorporeal thing, delivery is effected in the
b. Give the bidder a chance to change its bid within 24
following instances. Which should not be included?
hours
a. By the execution of a public instrument c. Submit to the Head of the Procuring Entity the bid
b. By mere consent or agreement of the contracting proposals of the bidder for his approval
parties d. Conduct another bidding for the bidder to qualify
c. By placing the titles of ownership in the possession
of the vendee 9. A sold B a specific thing. It was agreed that A would fix
d. By allowing the vendee to use his right as new the price a week later. At the agreed time, A set the
owner with the consent of the vendor. price at 10T. B agreed. Was the sale perfected?
a. No, because the price was left to the discretion of
4. S1: After the consummation of the merger agreement, one of the contracting parties
the notifying entities should notify the Philippine b. No, because at the time of sale, the price was not
Competition Commission of their merger agreement for fixed
its review and approval. c. Yes, because the price fixed by one of the parties
S2: Merger and Acquisition which reached the threshold was accepted by the other
amount that is not reported to PCC is null and void.
d. Yes, because there was agreement that A would fix
a. True, True the price
b. True, False
c. False, False 10. Which of the following is a not suspicious transaction?
d. False, True a. Melvin, a minimum wage earner, depositing
P300,000.00 in her account in BDO.
5. The following, except one, are some of the b. Ricky, a call center manager, deposits P100,000
circumstances where there is presumption of an three times weekly during the month of November.
equitable mortgage. Which should not be included? c. Jose, the pay-master of a construction company,
a. Price is unusually inadequate regularly deposits P300,000.00 in his bank account
b. Buyer remains in possession as lessee or otherwise that will be used to pay the salaries of the workers.
c. When upon or after the expiration of the right to d. Juan, who makes structured deposits in money in
repurchase another instrument extending the period the bank below P500,000, in order not to fall under
of redemption or granting a new period is executed the covered transaction of AMLA
d. Buyer retains for himself a part of the purchase price

Page 1 of 10 www.teamprtc.com.ph RFBT.1stPB5.23


TEAM PRTC

11. A tax law provides for payment of taxes on or before 16. The effects of a conditional obligation to give, once the
specified dates, otherwise delinquency penalties would condition has been fulfilled, shall retroact to the day of
be paid. Which of the following is correct? the constitution of the obligation. In obligations to do
a. Demand would be necessary to put the taxpayer in and not to do, who shall determine, in each case the
delay in the payment of the required taxes retroactive effect of the condition that has been
b. Demand would not be necessary to put the taxpayer complied with?
in delay in the payment of the required taxes a. Active subject
c. In case of delay for the payment of taxes, b. Passive subject
delinquency penalties would be paid c. Expert chosen by the parties
d. B and C d. Court
12. Statement 1: Covered institutions, when reporting 17. S1: The BSP and PDIC can examine the deposit
covered transactions to AMLC, are not deemed to have information of all depositors of the bank without the
violated the Bank Secrecy Law; however, they cannot consent of the deposits to determine the presence of
communicate the contents thereof, otherwise, they are unsafe bank practices.
criminally liable. S2. Courts cannot issue an order for examination of
Statement 2: A bank officer after having made a bank deposit without the consent of the depositor.
covered transaction report in the regular course of his S3. S1: In Bouncing Check Law, lack of criminal intent
work in the bank, reported the same to AMLC, and the is a defense.
person who owns the deposit account was exposed with a. All are true
criminal charges for kidnapping for ransom is not liable b. All are false
administratively, criminally or civilly. c. S1 I true, S2 and S3 are false
a. True, True d. S1 is true, S2 is false, S3 is true
b. True, False e. S1 is false, S2 and S3 are false
c. False, False
d. False, True. 18. Which of the following is not a rule in application of
payment?
13. Glicerio was appointed guardian of Sonny, the latter a. The debtor shall indicate the debt to which payment
being 11 years old, Sonny sold his parcel of land in must be applied after making payment.
writing to Bentong valued at P300,000 for P225,000 b. The creditor may make the designation by
suffering lesion by 1/4 the value. What is the status of specifying in the receipt which debt is being paid if
the contract? debtor does not apply payment
a. Rescissible c. If no application has been made or the application
b. Unenforceable is not valid, the debt which is most onerous to the
c. Enforceable debtor among those due, shall be deemed to have
d. Voidable been satisfied
d. If debts due are of the same nature and burden the
14. Statement 1: For a person to be liable for violation of payment shall be applied to all of them
BP22, the check must have been issued for payment or proportionately
guaranty of a pre-existing obligation.
Statement 2: A person can be charged with estafa and 19. S1: Subsequent compliance of the requirements of the
violation of BP 22 at the same time without violating the Truth in Lending Act will able the creditor/lender to
rule on Double Jeopardy. collect finance charges.
a. True, True S2. The finance charges must be made known to the
b. True, False debtor at the time the loan contract is perfected.
c. False, False a. True, True
d. False, True b. True, False
c. False, False
15. Hansel has the following accounts in the Landbank of d. False, True
the Philippines:
20. Juancho and Jovito owe P3,000 to Arturo, Antonio and
Account Name Type of Balance Martin. How much can Martin demand from Juancho?
per Bank Records Deposit a. P500
Hansel Trust Account 500,000.00 b. P1,000
Hansel Checking 100,000.00 c. P2,000
Account
d. P3,000
Hansel Savings 400,000.00
Deposit
21. A foreign corporation can engage in the insurance
Hansel Time Deposit 300,000.00
business in the Philippines, provided:
Hansel Flower Savings 100,000.00 a. total investment of a foreign insurance company in
Shop- Sole Deposit any registered enterprise shall not exceed twenty
Proprietor percent (35%) of the net worth of said foreign
insurance company nor twenty percent (35%) of the
Based on the table above, which statement is correct? capital of the registered enterprise, unless
a. The trust account is separately insured by the PDIC, previously authorized in writing by the
hence Hansel can claim the whole amount in case of Commissioner.
bank closure b. total investment of a foreign insurance company in
b. The Time deposit by Hansel is not covered by PDIC any registered enterprise shall not exceed twenty
c. Hansel has an uninsured deposit of P400,000.00 percent (30%) of the net worth of said foreign
d. None of the above insurance company nor twenty percent (30%) of the
capital of the registered enterprise, unless

Page 2 of 10 www.teamprtc.com.ph RFBT.1stPB5.23


TEAM PRTC

previously authorized in writing by the 27. D is the driver of a passenger bus which is owned and
Commissioner. operated by O. While D is driving the said bus, it met an
c. total investment of a foreign insurance company in accident through his negligence where P1, a passenger,
any registered enterprise shall not exceed twenty was injured an P2, a pedestrian, was also hurt. D is
percent (25%) of the net worth of said foreign liable to P2 for damages which arises from
insurance company nor twenty percent (25%) of the a. Law
capital of the registered enterprise, unless b. Quasi-contracts
previously authorized in writing by the c. Contracts
Commissioner
d. total investment of a foreign insurance company in
d. Delict
any registered enterprise shall not exceed twenty e. Quasi-delict
percent (20%) of the net worth of said foreign
insurance company nor twenty percent (20%) of the 28. Which of the following is not a characteristic of a joint
capital of the registered enterprise, unless indivisible obligation?
previously authorized in writing by the a. The creditor must proceed against all the debtors
Commissioner b. Each of the debtors can be held liable to pay for the
entire obligation
22. Conrad, a law student, bought law books at Rex c. If any of the debtors be insolvent, the others shall
Bookstore in credit terms. Conrad promised to pay the not be liable for his share
law books on July 30, 2022. On July 15, 2022, the d. Demand must be made to all the debtors
dormitory where Conrad is residing was burned because
the dormitory was struck by lightning. 29. Which of the following is an indivisible obligation?
a. Conrad is no longer liable to pay the books because a. Execution of a certain numbers of days
it has been lost through a fortuitous event. b. Accomplishment of work by metrical units
b. Conrad is still liable because the fire is not c. Susceptible of partial performance
considered a fortuitous event d. To give definite things
c. Conrad is not liable because Conrad was negligent
in stacking his law books at a dormitory 30. In payment by cession or when the debtor cedes or
d. Conrad is still liable because he has still the assigns his property to his creditor in payment of his
obligation to pay the money which cannot be debt, he shall be released from his obligation
extinguished a. To the full extent of the debtor’s obligations
b. Only to the extent of the net proceeds of the thing
23. Which of the following constitutes an innominate
assigned
contract?
c. Only to the extent allowed by the debtors
a. Michael sold to Martin a specific laptop computer in
exchange of a specific flat screen television.
d. Only to the extent allowed by the creditors
b. Elaine requested Monina for the latter to help the 31. A obliged himself to give B a car if B places among the
former in her assignment in Business Research. In
top ten in the CPA Board Exam. Subsequently, they
exchange, Monina requested Elaine for the latter to
agreed that a would give B the car if B merely passes
help the former in her assignment in Basic
the CPA Board Exam. This is an example of
Accounting. Both parties agreed.
a. Mixed novation
c. A offered to sell to B a car
b. Real novation
d. A and B agreed on the sale of Nicanor Reyes Street
in the amount of P10,000.00
c. Implied novation
d. Personal novation
24. The following are incapacitated to enter into a contract, e. Tacit novation
which is the exception?
a. Minors 32. A, B and C owe D, E and F P30,000.00. Suppose the
obligation was about to prescribe. F demanded payment
b. Insane or demented persons;
from A. What is the effect of the demand to the
c. Deaf mutes who do not know how to read and write; obligation?
d. Persons suffering from civil interdiction; or a. It will prejudice all their share in the obligation
e. Insolvent b. It will prejudice only the share of A
25. Which of the following agreements is unenforceable for
c. It will have a retroactive effect and will benefit D, E
and F.
failure to comply with the prescribed form?
a. Verbal agreement for payment of interest d. D, E and F will have a right of recourse against A
b. Oral authority to sell land 33. Statement 1 - The principle of quantum meruit is
c. Oral executed sale of land applicable to indivisible obligations.
d. Minor selling the land Statement 2 – Simplified Payment is the meeting in one
e. Verbal sale of land to take effect after one year person of the qualities of creditor and debtor with
respect to the same obligation.
26. If mistake, fraud, inequitable, conduct, or accident has Statement 3 – Conventional subrogation is effected with
prevented a meeting of minds of the parties, the proper the consent of the creditor at the instance of the old
remedy is ____________. debtor with the concurrence of the new debtor.
a. Reformation of the instrument a. All are true
b. Annulment of contract b. All are false
c. Ratification of contract c. 1 and 2 are true; 3 is false
d. Specific Performance d. 1 and 2 are false; 3 is true
e. Declaration of nullity of the contract e. 1 and 3 are false, 2 is true

Page 3 of 10 www.teamprtc.com.ph RFBT.1stPB5.23


TEAM PRTC

34. Amihan and Alena, both residents of Encantadia, are a. To refund the amount disbursed on behalf of the
very close friends. Amihan borrowed P20,000.00 from firm plus interest from the time the expenses were
Alena and through a promissory note, Amihan promised made.
to pay the loan “whenever her means permit”. However, b. To answer for obligation contracted in good faith in
the two were caught in a love triangle and as a result, the interest of the partnership.
became bitter enemies. Alena now demands payment of c. To answer for risks when only the use of specific and
the loan from Amihan because she is in dire need of determinate things is contributed to the
money and as a consequence of the love triangle, would partnership.
like to sue Amihan in case of failure to pay on the part
of the latter. In this case-
d. To answer for risks in consequence of its
management.
a. There being no period stipulated, the same is
immediately demandable. 40. L, B and C are partners. C is limited partner. Suppose
b. Amihan is in delay for non-payment of the monetary Jose Pidal is a creditor of the firm to the amount of
obligation P100,000.00, and after exhausting all the assets of the
c. Alena must bring action against Amihan for the partnership, there is a balance of P30,000.00. How can
purpose of asking the court to fix the period of Jose Pidal recover the P30,000.00?
payment. a. Jose Pidal must sue the firm and get the P30,000.00
d. Alena should file an action for collection for the sum from all the partners jointly.
of money b. Jose Pidal must sue the firm and get the P30,000.00
e. Alena cannot arbitrarily sue, as this will violate the from L and B
principle of unjust enrichment. c. Jose Pidal can recover the balance of P30,000.00
from any of the partners solidarily
35. On due date of A’s obligation, B his creditor, refused to d. None of the above
accept the full payment of the obligation. A deposited
the amount in court and notified B thereafter of the 41. Donald pledged his Singer Sewing Machine to Conrad
consignment. Was there valid tender of payment and for P8,000.00. Donald was unable to pay the obligation
consignation? 60 days after due date. Conrad sold the machine at
a. Yes, there was valid tender of payment and public auction for P6,000.00.
consignation a. Conrad cannot recover the deficiency of P2,000.00
b. No, it must have been made through compensation even if there is a stipulation that he can
c. No, there was no previous notice of consignation b. Conrad can recover the deficiency of P2,000.00
d. Yes there was valid confusion even without stipulation
e. None of the above c. Conrad cannot recover the deficiency of P2,000.00
d. Conrad can recover the deficiency of P2,000.00
36. Wilkins, Viva and Absolute formed a universal
partnership of profits. Wilkins contributed the use of his 42. Richard has in his name 1,000 shares of the capital stock
land. At the end of the partnership- of DEC Corporation as evidenced by stock certificate.
a. The land should be returned to Wilkins. Richard delivered the stock certificate to Ben who now
b. The land should be returned to Wilkins because claims to be the real owner of the shares, having paid
ownership had always been with him. Richard’s subscription. DEC refused to recognize and
c. The land should be returned to Wilkins as only the register Ben’s ownership. Which statement is correct?
use of the land was contributed and ownership had a. DEC Corporation’s refusal is justified because the
always been with him. certificate of stock was not duly indorsed by Richard
d. The land and its fruit should be returned to Wilkins to Ben and the transfer was not registered in the
as ownership had always been with him books of the corporation.
b. The refusal of DEC Corporation is not justified
37. Z Corp. amended its by-laws. The by-laws now requires because Ben is the one who paid the subscription
that every director of Z Corporation must own at least price and it would be unfair to Ben if Richard will
100 shares of stock in his name. receive the dividends.
a. The amendment is void because the Corporation c. The payment of the subscription by Ben is not valid
Code only requires at least (1) share of stock in the because he is not the subscriber, hence the
director’s name; corporation may treat Richard’s subscription unpaid
b. The amendment is valid because the by-laws merely d. Ben can compel the corporation to recognize the
provides for additional qualifications and did not do transfer and register the same in the corporate
away with the qualifications laid down by law; books because there is a voluntary delivery of the
c. The amendment is voidable. It is valid unless certificates of stock to him.
annulled by the court.
d. The amendment can be ratified by the majority vote 43. A, B, C, D, E and F are holders of non- voting shares of
of the OCS. FAR Inc. Subsequently, the Board of the corporation
adopted a proposal for the merger of FAR, Inc. With
38. The following, except one, are advantages of par value East, Inc. The corporate secretary then sent notice to all
shares of stock. Which one is the exception? holders of voting shares for a general stockholders’
a. Greater protection to creditors meeting the purpose of voting upon the proposal. A, B,
C, D, E and F were not notified, hence, were not able to
b. Liability of subscribers for unpaid subscription participate in the meeting where the merger was
c. Unlikelihood of distribution of dividends that are adopted.
ostensible profits
a. The failure of the corporation to notify A, B, C, D, E
d. Ease of sale and F and their non-inclusion in the meeting where
the merger was adopted is justified because they
39. Which of the following is not an obligation of the are not holders of voting share
partnership with every partner?

Page 4 of 10 www.teamprtc.com.ph RFBT.1stPB5.23


TEAM PRTC

b. The absence of notice and non-inclusion of A, B,C,D, d. The capitalist partners may exclude U from the
E and F in the said meeting is invalid because even partnership and ask indemnity for damages.
holders of non-voting share have the right to vote
on the issue of merger and consolidation of the 48. A, B, C, D, E and F entered into a contract of partnership
company with the purpose of operating a crocodile farm in
c. The absence of notice to A, B, C, D, E and F and Palawan. The following are their respective contributions
their non- inclusion in the meeting is insignificant to the capital: A =5%, B =55% C=5%, D =10% E
because the merger can still be adopted without =15% and F =10%. A and F were appointed managers
their approval to the proposal considering that without specification of their respective powers and
majority of the stockholders approved the same duties. A proposed a business expansion in Cebu but
d. Holders of non-voting shares of stocks are allowed was objected to by F. During the discussion of the
to vote on matters of dissolution only not on merger proposal at a partnership meeting, B sided with A while
and consolidation C, D and E sided with F.
a. A's proposal should be adopted because it was
44. L, M and N entered into contract of partnership for the approved by B who represents the controlling
operation of an employment agency. L contributed P1M interest.
cash and M a unit in a commercial building. N b. A's proposal should not be adopted because that is
contributed his industry being tasked to manage the the decision of the majority of the partners.
business. The contract was not accompanied with an c. Since A and B are the managing partners, they
inventory of the property contributed by M and it is not alone should decide on the matter. Since under the
in a public instrument. circumstances, there is disagreement between
a. The contract is void because there is no inventory of them, the same should be resolved by tossing of
the immovable property contributed attached to the coins.
said contract d. When important matters affecting the partnership
b. The contract is valid because a contract of are being decided upon like in the case at bar, the
partnership can be constituted in any form vote of the owner/s representing majority of
c. The contract of partnership is invalid because it was common fund should prevail.
not registered with the Securities and Exchange
Commission 49. In the election of the directors of ABA Corporation, A, B,
d. The contract is void because it is not in a public C, D, E, F and G were nominated. A, B, C, D and E
instrument, and there is no inventory of the received the highest number of votes and were
immovable property contributed attached to said proclaimed elected. F placed sixth with ten (10) votes
contract. lesser than E. Subsequently, E sold all his shares to G.
G claims now that he should sit in the Board because
45. Which of the following cannot qualify as a Limited the transfer of E's shares to him includes the latter's
Partnership position in the Board. F opposed the position of G
a. Partnership composed of an industrial partner, a claiming that he should be the one to sit in the Board
capitalist partner and a limited partner vacated by E because he was the sixth placer in the
election. Which statement is correct?
b. Partnership composed of 2 capitalist partners, and
an industrial partner a. The claim of G is correct because the position
attaches to the shares of stock once the holder
c. Partnership composed of no industrial partner, 3 thereof is elected to the Board and it can only be
capitalist partners and 2 limited partners
withdrawn by non-reelection.
d. Partnership composed of 2 capitalist partners, 2 b. The claim of F is correct because if one of the
industrial partners and a limited partner
members of the Board resigns or sells all his shares,
the seat vacated by him shall automatically vested
46. Which of the following is entitled first preference in
to loser with the highest number of votes.
payment in a general partnership during winding up:
c. The claim of G and F are both incorrect because E
a. Those owing to partnership creditors other than remains to be the director until his term expires.
partners
d. None of the above.
b. Those owing to the separate creditors of the
partners
50. A, B and C formed X partnership to engage in retail trade
c. Those owing to the partners other than for capital with a duration of ten years. Each contributed P100,000
and profits to the common fund. However, since A was appointed
d. Those owing to partners with respect to profits. the manager, he was given 60% of the profits while B
and C 20% each. Profit-sharing is made every end of
47. Q, R, S and T are capitalist partners while U is an the year. After ten years of existence of the partnership,
industrial partner in K partnership. The partnership is the partners continued the partnership affairs without
engaged in real estate business. Without the consent of their express consent with the same way it was
Q, R, S and T, partner U put up a construction supply habitually conducted to before the expiration of its
store to supply K partnership with construction period. Now, during the first profit-sharing, B and C
materials. Which of the following statements is not claim that the profits should be divided equally between
correct? the partners.
a. The capitalist partners may exclude U from the a. B and C's claim is legally tenable because the
partnership. partners equally contributed to the common fund;
b. The capitalist partners may avail themselves of the b. The claim of B and C is incorrect because the profit-
benefits which U may have obtained from the sharing should be the same the as the profit-sharing
operation of the construction supply. under the dissolved partnership;
c. The capitalist partners may choose to exclude U c. A should received the least profit to offset the
from the partnership and avail themselves of benefits derived from the partnership because A had
the benefits arising from the latter's operation of
construction supply.

Page 5 of 10 www.teamprtc.com.ph RFBT.1stPB5.23


TEAM PRTC

been receiving the biggest share under the 56. A bound himself to deliver to B a piece of land located
dissolved partnership; in Cagayan specifically described by Transfer Certificate
d. The partnership can not be continued bacause the of Title (TCT) No. 12345. Before delivery to A, the land
term thereof has expired. Hence, immediately upon bears fruits from a mango tree. In this case:
the expiration of the term, the parties should have a. the mango fruit shall belong to A
liquidated the partnership affairs. b. the mango fruit shall belong to B
c. the mango fruit shall belong either to A or B
51. A and B are partners in a certain business, A being the d. the mango fruit shall belong neither to A or B
manager. C owes A P5,000 and the partnership
P10,000, both debts are now due and demandable. C 57. Statement 1 - Vinculum juris refers to the juridical tie
pays A P3,000 and the latter issues a receipt under his that binds both the obligor and the obligee in an
name. obligation.
a. The P3,000 applies to A's personal credit alone Statement 2 - Advertisement for bidders is a form of
because he issued a personal receipt. contract and the advertiser is bound to accept the
b. The P3,000 shall be applied to the partnership credit highest or lowest bidder.
because the partnership's interest takes precedence a. If both statements are true
over that of a partner. b. if both statements are false
c. The P3,000 shall be divided equally between the and c. if first statement is true, second statement is false
A, that is, P1,500 for the firm and P1,500 for A.
d. if first statement is false, second statement is true
d. The P3,000 shall be applied proportionately to the
partnership credit and A's credit, that is, P2,000 to 58. Statement 1 - A bound himself to pay his creditor B,
the former and P1,000 to the latter. "whenever he is capable of paying the obligation." This
is not valid since the payment of the obligation depends
52. Which of the following is a ground for the judicial upon the sole will of the debtor.
dissolution of partnership? Statement 2 - The obligation is said to be alternative
a. Death of partner when the debtor binds himself to deliver a thing or its
b. Expulsion of a partner substitute.
c. Insolvency of a partner a. If both statements are true
d. Insanity of partner b. if both statements are false
c. if first statement is true, second statement is false
53. A, B and C entered into a partnership where C d. if first statement is false, second statement is true
contributed his apartment situated in Quiapo, Manila to
be converted into Office of the firm. Before the delivery 59. Statement 1 - Dolo causante refers to the fraud
of the title and possession of the said property to the committed by a person in performing his obligation.
partnership, a fire completely razed the said apartment. Statement 2 - In reformation, it will be proper when
What is the effect of the loss of the apartment? there was no meeting of minds between the contracting
a. The partnership is dissolved by reason thereof parties.
because C can no longer deliver the contribution he a. If both statements are true
has promised.
b. if both statements are false
b. The partnership subsists with the condition that C
must replace his contribution with a property
c. if first statement is true, second statement is false
equally satisfactory; d. if first statement is false, second statement is true
c. The partnership is not dissolved because the land
60. A is a minor. B is under guardianship. They entered into
where the apartment was situated remains.
a contract wherein A is bound to sell his car to B for
d. The partnership is not dissolved because from the P250,000. The status of the contract is:
moment C promised to contribute the apartment,
the partnership acquires ownership of the same and
a. Valid
thus, the loss is bome by the partnership b. Void
c. Rescissible
54. Which among the following is not a characteristic of d. Unenforceable
corporation? e. Voidable
a. It is created by operation of law.
b. It is automatically dissolved upon the death of all 61. A is indebted to B for P20,000.00. X is the guarantor of
stockholders. A. B is also indebted to A for P8,000.00. How much will
c. The shares of stocks are transferable. X be liable as guarantor, if A cannot pay?
d. The stockholders are liable only to the extent of a. P12,000.00
their investment. b. P20,000.00
c. P8,000.00
55. A, B and C formed a partnership to continue for a term d. P2,000.00
of five (5) years. On the third year, C sold his entire
interest to D. Which of the following is a right of D? 62. Which of the following agreements is void for failure to
a. D can ask for judicial rescission comply with the prescribed form?
b. D can exercise acts of management inasmuch as he a. verbal agreement for antichresis
is now a partner. b. verbal lease
c. D has a right to receive profits to which C would c. verbal contract of mutuum
otherwise be entitled. d. sale of real property which is not in a public
d. D can inspect the books of the partnership and ask instrument
for formal accounting. e. verbal commodatum

Page 6 of 10 www.teamprtc.com.ph RFBT.1stPB5.23


TEAM PRTC

63. A and B, minors, entered into a contract for the sale of a. Partnership property
the latter's car. When A's parents learned of the sale, b. Separate property of any general partner
they immediately conveyed to B's parents of their c. Both partnership property and separate property of
intention to ratify the same. B's parents, however, did a general partner cumulatively
not ratify the contract saying that the car has a
sentimental value to them. What is the effect of the
d. Both partnership property and separate property of
a general partner alternatively
ratification of A's parents to the contract?
a. No effect whatsoever e. Both partnership property and separate property of
a general partner successively
b. The contract became perfectly valid
c. It converted the contract into a voidable one 69. Pedro is a stockholder of a close corporation. Having
d. The contract is automatically set aside by reason of irreconcilable differences with his fellow stockholders,
the refusal of B's parents to ratify the contract he decided to withdraw from the said corporation. He
approaches you to exercise his right to withdraw, which
64. G was appointed guardian of S, the latter being 16 years is the ground he can use to withdraw?
old. S sold his parcel of land in writing to B valued at a. merger consolidation
P100,000 for P75,000 suffering lesion by ¼ of the value b. sale, lease, exchange, transfer, mortgage, pledge or
of the property. What is the status of the contract? other disposition of all or substantially all of the
a. Rescissible corporate property
b. Unenforceable c. amendment of articles of incorporation
c. Enforceable d. shortening corporate term
d. Void e. for any reason
e. Voidable
70. Berto offers to sell to Claro fifty sacks of white rice. Claro
65. In the dissolution of a limited partnership, which of the responds, “I agree to purchase fifty sacks only if the
following is the fourth in the priority in the distribution white rice is Grade A quality”. In this case
of partnership assets? a. A contract is perfected
a. Those to general partners in respect to profits b. Claro's statement is a counter-offer
b. Those to general partners in respect to capital c. Claro’s statement is an acceptance
c. Those to limited partners in respect to their share of d. Consensual contract to create to deposit
the profits and other compensation by way of
income on their contributions 71. The right to redeem by virtue of a conventional
d. Those to limited partners in respect to the capital of redemption, in the absence of an express agreement as
their contributions to the period shall last ____ from the date of the
e. Those to general partners other than for capital and contract
profits a. One year
b. Two years
66. Which of the following is a false statement? c. Four years
a. If a limited partnership will be created, registration d. 10 years
is a condition precedent for the creation of the
limited partnership
e. Six months
b. If one of the partners in a general partnership 72. The right of the mortgagor to redeem the mortgaged
contributed a building to the partnership, there property after his default in the performance of the
must be a public instrument and an inventory as a conditions of the mortgage but before the sale of the
requirement, otherwise the partnership is void mortgaged property is called -
c. If one of the partners contribute a specific car to the a. Equity of redemption
partnership, even if there is no appraisal of the car,
the partnership is valid
b. Legal redemption
d. If a limited partnership will be created, registration c. Conventional redemption
is a condition subsequent for the creation of the d. Right of pre-emption
limited partnership e. Right of redemption
e. If a limited partnership is not registered, a general
partnership is created 73. Deficiency judgment is allowed in the following
contracts, except:
67. Statement no. 1 - A stipulation which excludes one or a. Pledge
more of the partners from any share in the profits or b. Chattel mortgage
losses is void c. Real estate mortgage
Statement no. 2 - Any partner, as a general rule, shall d. Antichresis
have the rights to a formal account as to partnership
affairs 74. Allan sold to Bonnie a specific TV set for P5,000.00.
Statement no. 3 - Delectus personarum means as Bonnie made a partial payment of P2,000.00 and the
“much as the partnership deserves” balance to be paid three (3) months after. As a security
a. All are true for the obligation, Bonnie executed a chattel mortgage,
b. I and II are true; III is false Bonnie failed to pay and the TV set was foreclosed and
c. I and II are false; III is true was sold for only P2,000.00
d. II and III are false; I is true a. Under the Recto law, Bonnie is no longer liable for
e. I and III are false; II is true the deficiency of P1,000.00
b. Bonnie is still liable to pay the deficiency of
68. In a limited partnership, the creditor of a limited partner P1,000.00. The Recto law is not applicable because
may charge the interest of the indebted limited partner, the transaction is a straight term
the interest so charged may be redeemed with

Page 7 of 10 www.teamprtc.com.ph RFBT.1stPB5.23


TEAM PRTC

c. This is a case of dacion en pago. The obligation of B 79. C and D are opposing parties over a parcel of land
is extinguished owned by the State. To settle their dispute, they entered
d. Only the right to cancel the sale can be exercised into a compromise agreement whereby C recognized the
since there is non-payment of two or more ownership of D with a consideration. The compromise
installments agreement is:
a. Valid because it is mutually agreed by C and D
75. Penn sold to Quincy a Kenwood stereo set on September b. Valid because all the essential requisites of the
1, 2022. Penn sold to Robin the same Kenwood stereo contract are present
on September 5, 2022 and Robin immediately took c. Invalid because it is contrary to law
possession in good faith. Who has the better title to the d. Invalid because it is not in public instrument
Kenwood stereo set?
a. Quincy will be true owner as his contract of sale is 80. A owns an oil painting. In dire need of money, A sold
older than Robin’s contract of sale the painting to B for P2,000.00. After the sale, A
b. Robin is the true owner because he was the first who discovered that the painting was valuable and worth
took possession of the stereo set P10,000.00
c. Whoever pays first the purchase price a. A may rescind the contract on the ground of lesion
d. One who registers it in good faith or inadequacy of the price
b. A may rescind the contract on the ground of fraud
76. A was badly in need of money. He offered to sell his c. A may annul the contract on the ground of error
parcel of land to B for P100,000.00. B agreed and paid d. B is entitled to the benefit of the contract because it
A the P100,000.00 and signed a receipt. When B wanted is valid and binding
to register the sale, he needed a Deed of Absolute Sale.
What can B do? 81. A, B and C bound themselves solidarily to X in the
a. B may sue A to return the P100,000 under the legal amount of PhP12,000.00. The stipulations are as
maxim “No one shall enrich himself at the expense follows: A’s liability is due on August 30, 2022; B’s
of another liability is due on September 30, 2022 and C’s liability is
b. B may possess and utilize A’s land as a buyer in due on October 30, 2022. Thereafter, on October 30,
good faith 2022, X demanded payment from A and the latter
c. B may compel A to execute the Deed of Absolute immediately paid the entire obligation. On November 1,
Sale since the contract is valid 2022, X remitted the share of B, and C became
d. B cannot get back the P100,000 because the insolvent. To whom can A ask for the reimbursement of
contract is not enforceable the amount paid?
a. From B and C
77. FPJ Appliance Store is selling Starstruck vacuum b. From B or C
cleaners on a two-week trial. Ping asked the store to c. From B only
deliver him one vacuum cleaner. Which of the following d. From C only
is correct? e. From X
a. Ping becomes the owner of the vacuum cleaner
upon delivery 82. A and B are very close friends. A borrowed
b. The sale shall be absolute if Ping will not return the Php20,000.00 from B and through a promissory note, A
vacuum cleaner after two (2) weeks promised to pay the loan “whenever his means permit”.
c. Ping can return the vacuum cleaner to the store However, the two were caught in a love triangle and as
even if he finds nothing wrong with the quality of a result, became bitter enemies. B now demands
the said cleaner payment of the loan from A because he is in dire need
d. Ping has to pay the price of the vacuum cleaner if of money. In this case -
the vacuum cleaner is destroyed by a fortuitous a. There being no period stipulated, the same is
event immediately demandable
b. The consideration given for the loan is now
78. Mutuum is defined as extinguished, hence it is demandable
a. A contract whereby one of the parties delivers to c. A is in delay for non-payment of the monetary
another money and non-consumable things with the obligation
condition that the same amount of the same kind d. B must bring an action against A for the purpose of
and quality shall be paid asking the court to fix the period of payment
b. A contract whereby one of the parties delivers to e. B should file an action for collection for the sum of
another a property or other non-consumable thing money
with the condition that the same amount of the
same kind and quality shall be paid 83. A contract of sale was entered into by and between A
c. A contract whereby one of the parties delivers to and B where A sold his land and the improvements
another money or other consumable things with the thereon to B. The contract, however, did not state the
condition that the same amount of the same kind consideration given by B in exchange for the property of
and quality shall be paid A. A died and immediately thereafter, his heirs filed an
d. A contract whereby one of the parties delivers to action for reconveyance contending that the contract is
another money or other consumable things with the void due to lack of consideration. Is the action for
condition that another amount of different kind and reconveyance proper?
quality shall be paid a. Yes, because there was no consideration given in
e. A contract whereby one of the parties refuses to exchange
deliver to another money or other consumable b. No, because even if the cause is not stated in the
things with condition that the same amount of the contract, the same is presumed to exist and is lawful
same kind and quantity shall be paid unless the debtor proves otherwise

Page 8 of 10 www.teamprtc.com.ph RFBT.1stPB5.23


TEAM PRTC

c. Yes, the parties must the cause in the contract for it 88. Richard and Lucy have been living together for 10 years
to be valid without the benefit of marriage. During said period,
d. No, the contract is void. Richard sold to Lucy some of his properties. Give the
nature of the contract between Richard and Lucy.
84. X entered into a contract for the purchase of 2 lots from a. It is a valid sale because they are not married to
Y. Each lot costs Php 150,000.00. X already paid Php each other
90,000.00 each for each lot when his employment was b. It is void because the prohibition under the law
terminated on account of redundancy. X then failed to applies to couples living together without the benefit
pay for the balance. Due to this, Y filed an action for of marriage
rescission of both contracts contending that there was c. It is rescissible because it was undertaken in fraud
breach of X’s obligation. Will the action prosper? of creditors
a. Yes, because the power to rescind is implied in d. It is valid since it is not an illicit relationship
reciprocal obligation
b. Yes, based on the principle of Tacit Resolutory 89. Carmelita sells to Zenaida on June 1, 2022, 50 cases of
condition Bear brand milk with the agreement that Carmelita will
c. No, because rescission of contracts will result in immediately deliver them in Zenaida’s place in Cavite
unfairness. The total amount paid is sufficient to pay and payment of the purchase price to be made on
for the price of one lot hence it will be unfair to August 1, 2022. Then a week after delivery, Carmelita
rescind both learns that Zenaida has become insolvent. What is the
d. No, there was no agreement as to rescission. remedy of Carmelita?
a. She can exercise her possessory lien;
85. X and Y entered into a contract whereby Y’s services b. She can exercise her right of stoppage in transitu;
were engaged for the purpose of constructing a building c. Resell the merchandise
on the lot belonging to X. Z, Y’s friend, went X and told d. Demand payment immediately.
him that Y is well-known for this failure to finish his
contracts within the period agreed upon, prompting X to 90. The contract of mortgage between A and B as written
cancel his contract with Y. Can Z be held liable for stated the land object of the contract was sold by A,
damages? mortgagor, to B, mortgagee, with the right to
a. Yes, because third persons who induce another to repurchase. The remedy of A so that the mortgage
violate his contract can be held liable for damages contract will express their true intention is :
b. No, because X is within his rights to determine the a. Reformation of the contract
person who will construct the building on his land b. Cancellation of the contract
c. Yes, since Z was a party to the contract c. Enforce the contract without correction
d. No, there was no relativity of contract between them d. Accion Redhibitoria
e. Accion Quanti minoris
86. Horacio owes Roberto P150,000 secured by a mortgage
over the following properties: (a) A parcel of land 91. A realty company sold a house and lot to B in a
located in Manila valued at P150,000 and (b) a subdivision but they agreed that the company shall still
residential house and lot located in Cavite worth occupy the house and lot to serve as their temporary
P100,000. Horacio pays Roberto P80,000 leaving a office until their office building inside the subdivision is
balance of P70,000. Can Horacio compel Roberto to completed. B agreed. The house is deemed delivered
release the mortgage over the parcel of land located in thru:
Manila?
a. traditio longa manu
a. No, the principle of individuality of subscription will
apply in the collateral
b. traditio brevi manu
b. Yes, the consent of Roberto is not necessary c. tradition constitutum possesorium
because there will be partial extinguishment of the d. quasi-traditio
mortgage by operation of law;
92. B bought a specific appliance at a department store for
c. Yes, because the mortgage is divisible. Moreover,
a specific price on January 10, 2020, delivered on
the mortgage over the house and lot is more than
January 15, 2020. Assume that there is a hidden defect
enough to secure the payment of the balance;
in the thing sold so as to give rise to a valid action for
d. No, because a mortgage constituted to secure a breach of warranty against hidden defects or reduction
principal obligation is indivisible. Horacio cannot in the price. If you are B, you should file the action
demand for the partial extinguishment of the when:
mortgage until the debt is fully paid.
a. within 6 months from January 10, 2020
87. Hi-tech Corp. Sells a particular computer unit to Ismael b. within 6 months from January 15, 2020
for P40,000, properly described in an invoice with the c. after 6 months from January 10, 2020
indication in rubber stamp “Sale or Return, 15 days”. d. after 6 months from January 15, 2020
The computer unit was delivered to Ismael on June 1,
2022. The computer unit was destroyed by fortuitous 93. Today, X sold to Y his Honda CRV for P 400,000.00.
event on the 10th day after the said delivery. Unknown to both X and Y, the brother of X used the said
a. Hi-tech Corp. being the owner of the thing shall bear Honda CRV last night and completely destroyed the car
the loss in an accident. Is the contract of sale between X and Y
b. Ismael is not liable to pay the price because the valid?
thing was lost due to fortuitous event a. Yes, because it has all the essential elements of a
c. Ismael being the owner of the computer unit at the contract
time of the loss has to pay the price of P40,000 b. Yes, because it is already a perfected contact of sale
d. Both A and B c. No, because it lacks one of the essential elements
of a contact

Page 9 of 10 www.teamprtc.com.ph RFBT.1stPB5.23


TEAM PRTC

d. No, because the contract is already extinguished by d. Pactum De Non Alienando


fortuitous event
100. S1 – Right of redemption is allowed in chattel
94. AA owns 100 shares of stock in ABC Corporation. If mortgage.
there are five directors to be chosen, AA is entitled to S2 – For pledge to be valid to third persons, public
500 votes. AA gave the five candidates he wants to be document is required and the date and interest must
elected an equal 100 votes each. In this case, the voting be specified.
method used is: S3 – In real mortgage, Affidavit of Good Faith is
a. Straight voting method required to bind third persons
b. Cumulative voting for one candidate method S4 – In Legal pledge, the excess of the sale goes to
c. Cumulative voting by distribution method the debtor
a. Only S1 is false
d. Absentee voting b. Only S2 is true
c. Only S3 is false
95. In case of the adoption of the by-laws, it shall be
d. Only S4 is true
necessary to obtain the affirmative vote of:
e. All are false
a. The stockholders representing at least a majority of
the outstanding capital stock, or of at least a Thank you for participating in Team PRTC
majority of the members, in the case of non-stock Nationwide Online Open First Pre-Board
corporations Examination.
b. The stockholders representing at least 2/3 of the
outstanding capital stock, or of at least 2/3 of the
members, in the case of non-stock corporations
c. The 2/3 vote of the board of directors or trustees
d. The board of directors representing at least 3/4
96. A certificate of stock –
1st statement - is a written instrument signed by the
proper officer of corporation stating or acknowledging
that the person named therein is the owner of a
designated number of shares of its stock.
2nd statement - indicates the name of the holder, the
number, kind and class of shares represented, and the
date of issuance.
3rd statement - is merely an evidence of the
stockholder's interest in the corporation.
4th statement- is essential to make one a stockholder
in a corporation.
a. Only the 1st statement is false
b. Only the 2nd statement is false
c. Only the 3rd statement is false
d. Only the 4th statement is false

97. 1st statement - A representative suit is a suit brought


by one or more stockholders or members in the name
and on behalf of the corporation to redress wrongs
committed against it or to protect or vindicate corporate
rights, whenever the officials of the corporation refuse
to sue or, or are the ones to be sued or hold control of
the corporation.
2nd statement - A derivative suit exists when a
stockholder may bring a suit in behalf of himself and all
other stockholders who are similarly situated.
a. Only the 1st statement is true
b. Only the 2nd statement is true
c. Both of the statements are true
d. Both of the statements are false
98. The corporation shall furnish the stockholder or member
the most recent financial statement upon receipt of a
written request:
a. Within five (5) days
b. Within ten (10) days
c. Within fifteen (15) days
d. Within one (1) month
99. A stipulation prohibiting the mortgagor from selling the
mortgage property.
a. Pactum Reservati Dominii
b. Pactum Commissorium
c. Pactum Leonina

Page 10 of 10 www.teamprtc.com.ph RFBT.1stPB5.23


Excel Professional Services Inc.
Management Firm of Professional Review and Training Center (PRTC)
Online • Manila • Cavite • Laguna • Cebu • Cagayan De Oro • Davao
Since 1977

Financial Accounting and Reporting (FAR) OCAMPO/OCAMPO


FIRST PRE-BOARD EXAMINATION CPA Review February 19, 20 & 21, 2023

Multiple Choice. Select the letter that corresponds to the How much of these items would typically be reported as
best answer. This examination consists of 70 items and the inventories in the statement of financial position?
exam is good for three (3) hours. Good luck! a. P2,300,000 c. P2,220,000
b. P2,260,000 d. P2,000,000
1. It is a “global phenomenon” intended to bring about
transparency and a higher degree of comparability in
financial reporting, both of which will benefit the Use the following information for the next two questions.
investors and are essential to achieve the goal of one
An entity wholesales bicycles. It uses the perpetual
uniform and globally accepted financial reporting
inventory system. The entity's reporting date is Dec. 31. At
standards.
Dec. 1, inventory on hand consisted of 350 bicycles at P820
a. Norwalk agreement
each and 43 bicycles at P850 each. During the month of
b. World Trade
December, the following inventory transactions took place
c. Borderless accounting
(all purchase and sales transactions are on credit):
d. IFRS
Dec. 02 Sold 300 bicycles for P1,200 each.
2. Created by the IFRS Foundation to help meet the 03 Five bicycles were returned by a customer.
demand of international investors with global They had originally cost P820 each and were
investment portfolios for high quality, transparent, sold for P1,200 each.
reliable and comparable reporting by companies on 09 Purchased 55 bicycles at P910 each.
climate and other environmental, social and governance 13 Purchased 76 bicycles at P960 each.
(ESG) matters. 15 Sold 86 bicycles for P1,350 each.
a. International Financial and Sustainability Standards 16 Returned one damaged bicycles to the supplier.
Board This bicycle had been purchased on 9
b. International Sustainability Standards Board December.
c. International Financial Reporting Standards Board 22 Sold 60 bicycles for P1,250 each.
d. International Sustainability Standards Committee 26 Purchased 72 bicycles at P980 each.
29 Two bicycles, sold on 22 December, were
3. Presented below is a list of items that may or may not returned by a customer. The bicycles were
be reported as inventory in an entity’s Dec. 31 badly damaged so it was decided to write them
statement of financial position. off. They had originally cost P910 each.
a) Goods purchased FOB shipping point
4. The cost of goods sold for the month of December using
(in transit) P120,000
moving average method is (Round unit costs to the
b) Goods purchased FOB destination
nearest peso)
(in transit) 200,000
a. P367,230 c. P366,320
c) Freight charges on goods purchased 80,000
b. P365,410 d. P372,725
d) Materials on hand not yet placed
into production 350,000
5. The cost of goods sold for the month of December using
e) Factory supplies 20,000
FIFO method is
f) Office supplies 10,000
a. P367,230 c. P366,320
g) Interest cost incurred for inventories
b. P365,410 d. P372,725
that are routinely manufactured 40,000
h) Costs identified with units started
6. Techniques for the measurement of the cost of
but which are not yet completed 280,000
inventories may be used for convenience if the results
i) Costs identified with units completed
approximate cost. The following are acceptable for year-
but not yet sold 310,000
end financial reporting purposes, except
j) Goods out on consignment at
a. Standard cost method
another company’s store 800,000
b. Retail method
k) Goods held on consignment from
c. Gross profit method
another company 450,000
d. None of these
l) Goods sold on installment basis 100,000
m) Goods sold to another company, for
7. Compute for the cost of inventory lost in fire using the
which the company has signed an
data below:
agreement to repurchase at a set
price that covers all costs related to Inventory, Jan. 1 P 51,600
the inventory 300,000 Purchases 368,000
n) Goods sold FOB seller (in transit) 120,000 Sales 583,000
o) Goods sold FOB buyer (in transit) 40,000 Purchase returns 11,200
p) Costs incurred to advertise goods 20,000 Purchase discounts taken 5,800
held for resale Freight in 3,800
q) Securities acquired for the purpose Sales returns 8,600
of selling in the near term 500,000
r) Equipment held for sale in 80,000
accordance with PFRS 5
s) Cryptocurrencies 180,000

Page 1 of 8 www.teamprtc.com.ph FAR.1stPB5.23


TEAM PRTC

A fire destroyed the entire inventory except for 12. Bearer plant is carried at
purchases in transit, FOB shipping point, of P2,000 and a. Its cost less any accumulated depreciation and any
goods having selling price of P4,900 that were salvaged accumulated impairment losses.
from the fire. The average gross profit rate on net sales b. Its fair value at the date of the revaluation less any
is 40%. subsequent accumulated depreciation and
a. P59,760 c. P62,660 subsequent accumulated impairment losses.
b. P56,940 d. P56,820 c. Its fair value less costs to sell.
d. Either a or b.
8. The records of an entity report the following data for the
month of January: 13. Which of the following plants will likely qualify as bearer
Beginning inventory at cost P 440,000 plants and accounted for in accordance with PAS 16?
Beginning inventory at sales price 800,000 I. Strawberry vines
Purchases at cost 4,500,000 II. Watermelon vines
Initial markup on purchases 2,900,000 III. Kiwi vines
Purchase returns at cost 240,000 IV. Cucumber vines
Purchase returns at sales price 350,000
Freight on purchases 100,000 a. I, II, III and IV c. I and III only
Additional mark up 250,000 b. I, III and IV only d. None of them
Mark up cancellations 100,000
Mark down 600,000 14. An entity reported property, plant and equipment of
Mark down cancellations 100,000 P20,000,000 in its statement of financial position as of
Sales 5,300,000 Dec. 31, 2023. This amount includes the following:
Sales allowances 300,000 • Land held for undetermined future use,
Sales returns 400,000 P3,000,000.
Employee discounts 200,000 • Property occupied by employees, P8,000,000. The
Theft and other losses 100,000 employees pay rent at market rates.
Using the average retail inventory method, the entity’s • Equipment held to earn rentals under operating
ending inventory at cost is lease, P1,200,000.
a. P1,024,000 c. P1,536,000 Compute for the adjusted property, plant and
b. P1,472,000 d. P1,664,000 equipment.
a. P 9,000,000 c. P17,000,000
b. P15,800,000 d. P17,800,000
Use the following information for the next two questions.
An entity is engaged in agricultural activity. Its trial balance 15. When determining the commercial substance of the
at Dec. 31 presents the following assets related to its exchange, which of the following items is not
farmland: considered?
• Two tractors (P500,000 each) a. Cash flow of exchanged asset.
• Four computers (P25,000 each) b. Cash flow of new asset.
• Computer software (P50,000) c. Cash flow from tax effects on the exchange to avoid
• Fruit-bearing trees (estimated value, P20 million of 
taxes.
which P3 million is attributed to the fruits attached to d. Cash flow from potential sale of new equipment at
the trees). 
a later date.
• Harvested fruits (estimated value, P2 million)
• Trees grown for use as lumber (estimated value, P10 16. In accordance with PIC Q&A No. 2012-02, it is
million) appropriate for an entity to account for the carrying
• Trees that are cultivated both for their fruit and their value of the old building as part of the cost of the new
lumber (estimated value, P8 million) building (constructed for rental to others), in which of
• Maize and wheat (estimated value, P4 million) the following?
a. An entity acquired a piece of land with existing
9. How much should be accounted for as biological assets? building with the intention to demolish the old
a. P25 million c. P7 million building right away in order to construct a new
b. P17 million d. P3 million building on its site as part of its planned
redevelopment.
10. How much should be accounted for as property, plant b. An entity acquired a piece of land with existing
and equipment? building with the intention to initially use the old
a. P17.525 million c. P26.100 million building as an owner-occupied property and then
b. P18.100 million d. P30.100 million demolish it in a future period and replace it with a
new building.
11. Which of the following is not an agricultural activity? c. Both a and b.
a. Pharma company growing its own plants in order to d. Neither a nor b.
produce drugs.
b. Dairy company grows its own bacteria and cultures 17. An entity has the following items of Machinery at Dec.
and then adds them to its yogurts. 31, 2023:
c. An entity operating zoo with an active program of Machine No. Cost Acc. Dep.
breeding animals.
1 P100,000 P95,000
d. An entity catching fish in the ocean in relation to its
sardines business. 2 200,000 160,000
3 300,000 210,000
4 400,000 240,000

Page 2 of 8 www.teamprtc.com.ph FAR.1stPB5.23


TEAM PRTC

Additional information: 22. Cute Corporation owns the following properties at Jan.
• All items – useful is 10 years and the fair value is 1, 2023:
higher than the carrying amount at Dec. 31, 2023
Property A
• Machine No. 2 - idle during 2023
• Machine No. 3 - retired from active use on June 1, An office building used by Cute for administrative
2023 but not yet derecognized at Dec. 31, 2023 purposes with a depreciated historical cost of P2 million.
• Machine No. 4 - classified as held for sale in At Jan. 1, 2023 it had a remaining life of 20 years. After
accordance with PFRS 5 on July 1, 2023 but still a re-organization on July 1, 2023, the property was
unsold at Dec. 31, 2023 leased to a third party and reclassified as an investment
property applying Cute’s policy of the fair value model.
The total depreciation for the year 2023 is An independent valuer assessed the property to have a
a. P95,000 c. P75,000 fair value of P2.3 million at July 1, 2023, which had risen
b. P80,000 d. P57,500 to P2.34 million at Dec. 31, 2023.
18. An entity takes a full year's depreciation expense in the Property B
year of an asset's acquisition, and no depreciation Another office building sub-leased to a subsidiary of
expense in the year of disposition. Data relating to one Cute. At Jan. 1, 2023, it had a fair value of P1.5 million
of the entity’s depreciable assets at Dec. 31, 2022, are which had risen to P1.65 million at Dec. 31, 2023. At
as follows: Jan. 1, 2023 it had a remaining life of 15 years.
Acquisition year 2020
In relation to these properties, determine the net
Cost P110,000
amount to be recognized in profit or loss in the entity’s
Residual value 20,000
separate financial statements for the year ended Dec.
Accumulated depreciation 72,000
31, 2023.
Estimated useful life 5 years
a. P140,000 c. P490,000
Using the same depreciation method as used in 2020,
b. P190,000 d. P540,000
2021, and 2022, how much depreciation expense should
the entity record in 2023 for this asset?
23. An entity acquired a broadcasting license for
a. P12,000 c. P22,000
P60,000,000. The license is renewable every 10 years if
b. P18,000 d. P24,000
the entity provides at least an average level of service
to its customers and complies with the relevant
legislative requirements. The license may be renewed
Use the following information for next two questions.
indefinitely at little cost and has been renewed twice
An entity acquired a building on Jan. 1, 2019 at a cost of before the most recent acquisition. The acquiring entity
P50,000,000. The building has an estimated life of 10 years intends to renew the license indefinitely and evidence
and residual value of P5,000,000. The building was revalued supports its ability to do so. Historically, there has been
on Jan. 1, 2023 and the revaluation revealed replacement no compelling challenge to the license renewal.
cost of P80,000,000, residual value of P2,000,000 and
At the time of acquisition, the technology used in
revised total life of 12 years.
broadcasting is not expected to be replaced by another
technology at any time in the foreseeable future.
19. The carrying amount of building as of Dec. 31, 2023 is
Therefore, the license is expected to contribute to the
a. P28,250,000 c. P42,950,000
entity’s net cash inflows indefinitely. The license is valid
b. P42,700,000 d. P48,800,000
for six years before the next renewal.
20. The revaluation surplus as of Dec. 31, 2023 is On Jan. 1, 2023, the licensing authority decides that it
a. P14,000,000 c. P15,400,000 will no longer renew broadcasting license, but rather will
b. P14,700,000 d. P16,800,000 auction the license. At the time the licensing authority’s
decision is made, the entity’s broadcasting license has
21. Quirino, Inc. and its subsidiaries have provided you, three years until it expires. The entity expects that the
their PFRS specialist, with a list of the properties they license will continue to contribute to net cash inflows
own: until the license expires.
• Land held by Quirino, Inc. for undetermined future
The carrying amount of the license at Dec. 31, 2023
use, P5,000,000.
should be
• A vacant building owned by Quirino, Inc. and to be
a. Nil c. P40,000,000
leased out under an operating lease, P20,000,000.
b. P20,000,000 d. P60,000,000
• Property held by a subsidiary of Quirino, Inc., a real
estate firm, in the ordinary course of its business,
P30,000,000.
Use the following information for the next three questions.
• Property held by Quirino, Inc. for use in production,
P1,000,000. Entity O purchased a patent from an inventor immediately
• A hotel owned by Sugo, Inc., a subsidiary of Quirino, after it was registered. The asking price is P1,980,000. The
Inc., and for which Sugo, Inc. provides security entity paid for the patent as follows: cash, P720,000;
services for its guests’ belongings, P50,000,000. issuance of 18,000 shares of its own ordinary shares, par
• A building owned by Quirino, Inc. being leased out P10 (fair value, P20 per share); and a note payable due at
to Status, Inc, a subsidiary of Quirino, Inc., the end of three years, face amount, P900,000, noninterest-
P20,000,000. bearing. The present value of this note using an appropriate
discount rate is P620,000.
How much will be reported as investment properties in
Quirino, Inc. and its subsidiaries consolidated financial
The entity believes that the patent will generate future
statements?
economic benefits evenly over its legal life.
a. P25,000,000 c. P75,000,000
b. P45,000,000 d. P95,000,000
After acquisition, the following transactions and events
occurred:

Page 3 of 8 www.teamprtc.com.ph FAR.1stPB5.23


TEAM PRTC

• At the beginning of the second year, the entity paid more than would be expected as a result of the
P200,000 to successfully defend the patent in an passage of time or normal use.
infringement suit. b. Significant changes with an adverse effect on the
• At the beginning of the fourth year, the entity determined entity have taken place during the period, or will
that the remaining estimated useful life of the patent is take place in the near future, in the technological,
only five years. market, economic or legal environment in which the
• Paid annual fee of: entity operates or in the market to which an asset
Fifth year – P3,240 is dedicated.
Sixth year – P4,320 c. Market interest rates or other market rates of return
• At the end of the sixth year, the entity sold the patent on investments have increased during the period,
for P610,000 and paid P12,000 for disposal costs. and those increases are likely to affect the discount
rate used in calculating an asset’s value in use and
24. The carrying amount of the patent at the end of year 1 decrease the asset’s recoverable amount materially.
is d. Evidence is available of obsolescence or physical
a. P1,530,000 c. P1,700,000 damage of an asset.
b. P1,615,000 d. P1,881,000
32. An entity is testing an asset for impairment. The
25. The carrying amount of the patent at the end of year 4 carrying amount of the asset is P85,000. The following
is data has been obtained by the entity in relation to the
a. P 952,000 c. P1,139,158 asset.
b. P1,108,800 d. P1,156,000 • Future cash flows expected to be derived from the
asset, P100,000.
26. The gain or loss on disposal at the end of year 6 is • Estimated fair value of the asset, P80,000.
a. P20,000 c. P 32,000 • Present value of future cash flows expected to be
b. P28,421 d. P122,000 derived from the asset, P60,000.
• Costs of disposal for the asset, P2,000.
27. These cryptographic assets give holders no ownership
How much should be recognized as impairment loss?
in a company’s platform or assets and, although they
a. Nil c. P 7,000
might be traded between holders, they are not primarily
b. P5,000 d. P25,000
used as a medium of exchange.
a. Cryptocurrencies
b. Asset-backed tokens
Use the following information for next two questions.
c. Utility tokens
d. Security tokens An entity reported an impairment loss of P250,000 in its
income statement for the year 2020. This loss was related
28. Cryptocurrencies can be classified as to an item of property, plant and equipment which was
a. Financial assets at fair value through profit or loss acquired on Jan. 1, 2012 with a cost of P2,000,000.
b. Financial assets at fair value through other Depreciation on the asset is computed on a straight-line
comprehensive income basis and annual depreciation on cost is P80,000.
c. Financial assets at amortized cost Depreciation for the year 2021 was computed on the asset’s
d. None of these recoverable amount at Dec. 31, 2020. On Dec. 31, 2023,
the entity decided to measure the asset using revaluation
29. An entity purchased in 2021 a property that contained model. This asset was then appraised at a fair value of
mineral deposit for P4,500,000. Estimated recovery was P1,650,000.
1,000,000 metric tons of deposits. Development costs
of P150,000 were also incurred in the same year. The 33. The gain on impairment recovery to be recognized by
mining property was expected to be worth P600,000 the entity in 2023 profit or loss is
after the mineral deposits had all be removed. During a. Nil c. P218,125
2022, the company extracted and sold 100,000 metric b. P203,125 d. P250,000
tons of minerals. Further development costs of P75,000
were incurred in 2023, and the estimate of total 34. The revaluation increase to be recognized by the entity
recoverable deposits (including the amount extracted in in 2023 other comprehensive income is
2022) was revised to 925,000 metric tons. During a. P530,000 c. P610,000
2023, the company recovered 150,000 metric tons. b. P563,125 d. P748,125
The depletion for the year 2023 is
35. An entity accounts for non-current assets using the cost
a. P603,658 c. P676,500
model. On Oct. 30, 2023, the entity classified a non-
b. P618,750 d. P750,000
current asset as held for sale in accordance with PFRS 5.
At that date the asset's carrying amount was
30. PAS 36 does not apply to which of the following assets?
P15,000,000, its fair value was estimated at P11,000,000
a. Investment in associate
and the costs to sell at P1,500,000. On Nov. 20, 2023 the
b. Investment property that is measured using the
asset was sold for net proceeds of P9,200,000.
cost model
c. Property, plant and equipment that is measured In accordance with PFRS 5, what amount should be
using the revaluation model included as a loss on disposal in the entity’s statement of
d. Biological assets measured at fair value less costs to comprehensive income for the year ended Dec. 31, 2023?
sell a. Nil c. P5,500,000
b. P300,000 d. P5,800,000
31. Internal sources of information indicating that an asset
may be impaired include 36. An entity accounts for non-current assets using the
a. There are observable indications that the asset’s revaluation model. On June 30, 2023, the entity classified
value has declined during the period significantly a freehold property as held for sale in accordance with
PFRS 5. At that date the property's carrying amount was

Page 4 of 8 www.teamprtc.com.ph FAR.1stPB5.23


TEAM PRTC

P290,000 and the balance on the revaluation reserve was • Check written and dated Dec. 29, 2023 and
P20,000. At that date its fair value was estimated at delivered to payee on Jan. 2, 2024, P200,000.
P330,000 and the costs to sell at P20,000. At Dec. 31, • Check written on Dec. 27, 2023, dated Jan. 2,
2023, the property's fair value was estimated at 2024, delivered to payee on Dec. 29, 2023,
P325,000 and the costs to sell at P25,000. P300,000.
• Equity investment held for trading expected to be
The balance of the entity’s revaluation reserve as of Dec.
sold within 3 months, P600,000.
31, 2023 is
a. Nil c. P40,000 Compute for the adjusted cash and cash equivalents.
b. P30,000 d. P60,000 a. P4,780,000 c. P5,330,000
b. P5,280,000 d. P5,380,000
37. Bataan Inc. was granted a parcel of land by a local
government authority. The condition attached to this 41. An entity reported cash and cash equivalents of
grant was that Bataan Inc. should clean up this land and P12,000,000 in its statement of financial position as of
lay roads by employing laborers from the village in Dec. 31, 2023. This amount includes the following:
which the land is located. The entire operation will take • Customer’s check for P100,000 returned by bank
three years and is estimated to cost P100 million. This on Dec. 29, 2023 due to insufficient fund but
amount will be spent in this way: P20 million each in the subsequently redeposited and cleared by the bank
first and second years and P60 million in the third year. on Jan. 3, 2024.
The fair value of this land is currently P120 million. How • Customer’s check for P200,000 dated Jan. 2,
much should be recognized as income from government 2024, received on Dec. 29, 2023.
grant at the end of the first year? • Cash earmarked for bonds payable due on June
a. Nil c. P24,000,000 30, 2024, P5,000,000.
b. P20,000,000 d. P40,000,000 • P1,000,000 of compensating balance against
short-term borrowing arrangement at Dec. 31,
38. An entity started construction on a building on Jan. 1 of 2023. The compensating balance is legally
the current year and completed construction on Dec. 31 restricted as to withdrawal.
of the same year. Aries had only two interest notes • Check written and dated Dec. 29, 2023 and
outstanding during the year, and both of these notes delivered to payee on Jan. 2, 2024, P500,000.
were outstanding for all 12 months of the year. The • Check written on Dec. 27, 2023, dated Jan. 2,
following information is available: 2024, delivered to payee on Dec. 29, 2023,
Average accumulated expenditures P250,000 P800,000.
Ending balance in construction in • One-year certificate of deposit, P2,000,000.
progress before capitalization of Compute for the adjusted cash and cash equivalents.
interest 360,000 a. P3,700,000 c. P 8,700,000
6 percent note incurred specifically b. P5,700,000 d. P10,000,000
for the project 150,000
9 percent long-term note 500,000
What amount of interest should the entity capitalize for Use the following information for next two questions.
the current year? An entity had the following bank reconciliation on June 30:
a. P15,000 c. P22,500
b. P18,000 d. P27,900 Balance per bank statement, June 30 P3,000,000
Add: Deposit in transit 400,000
39. Which statement is incorrect? Total 3,400,000
a. In accordance with PIC Q&A No. 2018-10, an entity Less: Outstanding checks 900,000
should disclose write-downs of inventory held at the Balance per book, June 30 P2,500,000
end of the reporting period.
b. In accordance with PIC Q&A No. 2012-02, the costs The bank statement for the month of July showed the
incurred in relation to demolition (or the physical following:
tearing down) of the old building to give way for the Deposits (including P200,000 note
construction of the replacement building should collected for the entity) P9,000,000
preferably be capitalized as part of the cost of the Disbursements (including P140,000
new building. NSF check and P10,000 service
c. In accordance with PIC Q&A No. 2017-6, collector’s charge) 7,000,000
items for administrative or aesthetic purposes can be
accounted for in accordance with PAS 40. All reconciling items on June 30 cleared through the bank in
d. Subsequent expenditure on brands, mastheads, July. The outstanding checks totaled P600,000 and the
publishing titles, customer lists and items similar in deposits in transit amounted to P1,000,000 on July 31.
substance (whether externally acquired or internally
generated) is always recognized in profit or loss as 42. What is the amount of cash receipts per books in July?
incurred. a. P8,400,000 c. P9,600,000
b. P9,400,000 d. P9,800,000
40. An entity reported cash and cash equivalents of
P7,000,000 in its statement of financial position as of 43. What is the amount of cash disbursement per books in
Dec. 31, 2023. This amount includes the following: July
• Cryptocurrencies, P550,000. These are not held a. P5,950,000 c. P6,700,000
for sale in the ordinary course of business nor for b. P6,550,000 d. P7,450,000
investment purposes.
• Fund for acquisition of equipment, P800,000. 44. When the allowance method of recognizing uncollectible
• Cash surrender value of life insurance policy, accounts is used, the entry to record the write-off of a
P100,000. specific account
• Customer’s check for P170,000 dated Jan. 2,
2024, received on Dec. 29, 2023.

Page 5 of 8 www.teamprtc.com.ph FAR.1stPB5.23


TEAM PRTC

a. Decreases both accounts receivable and the The carrying amount of the note receivable of the entity
allowance for uncollectible accounts. as of Dec. 31, 2023 is
b. Decreases accounts receivable and increases the a. P365,802 c. P420,154
allowance for uncollectible accounts. b. P407,547 d. P500,000
c. Increases the allowance for uncollectible accounts
and decreases net income. 49. The following information pertains to Lender A’s loan
d. Decreases both accounts receivable and net income. portfolio at Dec. 31, 2023:
PV of Expected
45. Your analysis of the accounts receivable of an entity Future Cash Past due
indicates the following: Loan Amount Flows status
Accounts receivable, Jan. 1 P 300,000 1 P600,000 P360,000 91 days
Allowance for doubtful accounts, Jan. 1 40,000 2 500,000 450,000 Current
Credit sales during the year 1,200,000 3 400,000 320,000 31 days
Cash collections during the year 1,100,000 4 300,000 270,000 Current
Accounts receivable written off during 5 200,000 160,000 61 days
the year 20,000 6 100,000 60,000 91 days
In prior years, the entity’s bad debt expense has Lender A considers all loans over 90 days past due to be
averaged 2% of credit sales. credit-impaired based on historical experience with
On Dec. 31, what would be the amount of the entity’s recovering the associated debt.
accounts receivable, net of any allowance for doubtful Additional information taking into account historical
accounts, assuming that the entity uses the credit sales information, current conditions and forward- looking
method to estimating bad debt expense? information, including actual loss experience and
a. P336,000 c. P360,000 recoveries from the sale of collateral, is as follows:
b. P358,000 d. P400,000
Probability of default in the next 12 months 2%
46. An entity estimates bad debt expense at ½% of credit Lifetime probability of default
sales. The entity reported accounts receivable and Credit-impaired loans 100%
allowance for uncollectible accounts of P471,000 and Not credit-impaired loans 5%
P1,650 respectively, at Dec. 31, 2022. During 2023,
entity’s credit sales and collections were P315,000 and The total loss allowance to be recognized by Lender A at
P319,000, respectively, and P1,720 in accounts Dec. 31, 2023 is
receivable were written off. The balance of entity’s a. P280,000 c. P283,600
accounts receivable at Dec. 31, 2023 is b. P282,400 d. P287,600
a. P465,280 c. P469,280
b. P467,000 d. P473,280 50. As defined in PFRS 9, lifetime expected credit losses are
a. The expected credit losses that result from all
47. An entity sold equipment on July 1, 2022. The possible default events over the expected life of a
equipment cash price is P79,000. The buyer signed a financial instrument.
deferred payment contract that provides for a down b. The weighted average of credit losses with the
payment of P10,000 and an 8-year note for P103,472. respective risks of a default occurring as the
The note is to be paid in 8 equal annual payments of weights.
P12,934. The payments include 10% interest and are c. All cash shortfalls discounted at the original effective
made on June 30 of each year, beginning June 30, 2023. interest rate (or credit-adjusted effective interest
rate for purchased or originated credit-impaired
The total interest income for the year ended Dec. 31, financial assets).
2023 is d. The credit losses on
assets that are forecast to
a. P5,982 c. P6,612 actually default in the next 12 months.
b. P6,599 d. P6,900
51. An entity often factors its accounts receivable. The
48. On Jan. 1, 2019, an entity sold a machine with a finance company requires an 8% reserve and charges a
carrying amount of P300,000 and accepted in exchange 1.5% commission on the amount of the receivable. The
a promissory note with a face value of P500,000, a due remaining amount to be advanced is further reduced by
date of Dec. 31, 2028, and a stated rate of 4%, with an annual interest charge of 16%. What proceeds
interest receivable at the end of each year. The fair (rounded to the nearest peso) will the enterprise receive
value of the machine is not readily determinable and the from the finance company at the time a 110,000
note is not readily marketable. Under the account that is due in 60 days is turned over to the
circumstances, the note is considered to have an finance company?
appropriate imputed rate of interest of 8%. a. P83,630 c. P99,550
b. P81,950 d. P96,895

52. Which statement is correct regarding accounting for


transfers of receivables in accordance with PFRS 9?
a. The transfer of risks and rewards evaluated is
evaluated by determining the transferee’s ability to
sell the asset.
b. A sale of a financial asset together with a total
return swap that transfers the market risk exposure
back to the entity is an example of a transfer that
qualifies for derecognition.

Page 6 of 8 www.teamprtc.com.ph FAR.1stPB5.23


TEAM PRTC

c. The entity shall determine whether it has retained 58. On Apr. 1, 2023, the entity purchased 25,000 ordinary
control of the financial asset if an entity neither shares of Brian Corp. at P180 per share which reflected
transfers nor retains substantially all the risks and book value as of that date. At the time of the purchase,
rewards of ownership of a transferred asset. Brian had 100,000 ordinary shares outstanding. The
d. The entity shall continue to recognize the entity paid transaction costs of P67,500. The first
transferred asset in its entirety if an entity neither quarter statement ending Mar. 31, 2023 of Brian
transfers nor retains substantially all the risks and recorded profit of P480,000. For the year ended Dec.
rewards of ownership of a transferred asset, and 31, 2023, Brian reported profit of P2,400,000. Brian
retains control of the transferred asset paid the entity dividends of P60,000 on June 1, 2023
and again P60,000 on Dec. 31, 2023. The shares of
53. Which of the following returns is not consistent with Brian are selling at P190 per share on Dec. 31, 2023.
cash flows that are solely payments of principal and
The carrying amount of the investment in Brian Corp. as
interest on the principal amount outstanding?
of Dec. 31, 2023 should be
a. Return for the time value of money and credit risk
a. P4,860,000 c. P4,980,000
b. Return for liquidity risk
b. P4,927,500 d. P5,047,500
c. Return for amounts to cover expenses and a profit
margin
59. On Jan. 1, 2023, an entity acquired 25% of the shares
d. Return for equity price risk
of an investee for P425,000. At this date all the
identifiable assets and liabilities of the investee were
recorded at amounts equal to fair value, and the equity
Use the following information for next two questions.
of investee consisted of the following:
On April 1, 2021, the entity purchased 5-year P1,000,000 Share capital P1,000,000
10% bonds of June Corp. The bonds are dated Jan. 1, 2021. General reserve 300,000
The bonds were purchased to yield 8%. Interest is payable Asset revaluation surplus 200,000
annually every Dec. 31. The entity uses the ‘held for Retained earnings 200,000
collection’ business model for acquired and originated debt
instruments. The issuer paid the interest as scheduled in In 2023, investee reported profit of P250,000. P50,000
2021 and 2022. During 2023, the issuer of the bonds is in of the asset revaluation surplus was realized in 2023.
financial difficulties and it becomes probable that the issuer Investee paid a P40,000 dividend and transferred
will be put into administration by a receiver. On Dec. 31, P30,000 to general reserve. What is the carrying
2023, the entity estimated that none of the interest will be amount of the investment as of Dec. 31, 2023?
collected and only P800,000 of the principal will be collected a. P477,500 c. P465,000
on maturity date. b. P490,000 d. P482,500

54. The required loss allowance on investment in June Corp. 60. An entity has the following accounts with an investee,
bonds at Dec. 31, 2023 is an associate:
a. P314,160 c. P365,721 Investment in ordinary shares P4,500,000
b. P349,846 d. Nil Investment in preference shares 1,600,000
Loans receivable - unsecured 900,000
55. How much is the interest income to be recognized in Loans receivable - secured 500,000
2024 on investment in June Corp. bonds? Accounts receivable 200,000
a. P84,125 c. P54,867 Accounts payable 100,000
b. P82,855 d. Nil
If the ‘share of loss of associate’ recognized by the entity
56. On July 1, 2023, TGV purchased 10,000 of BPO’s 50,000 is P6,500,000, how much should be allocated to Loans
outstanding shares at a price of P6.00 per share. BPO receivable – unsecured?
had earnings of P3,000 per month during 2023 and paid a. Nil c. P720,000
dividends of P10,000 on Mar. 1, 2023 and P12,500 on b. P400,000 d. P900,000
Dec. 1, 2023. The fair value of BPO’s shares was P6.50
per share on
Dec. 31, 2023. 61. Which statement is correct regarding accounting for
Assuming that TGV accounts for its investment in BPO derivatives?
as a held-for-trading investment, what would be the a. Derivative assets can be classified as financial
total effect on TGV’s profit or loss for the year ended assets at fair value through OCI.
Dec. 31, 2023? b. An entity may reclassify derivatives out of fair value
a. P2,500 c. P6,500 through profit or loss when, and only when, it
b. P4,500 d. P7,500 changes its business model for managing financial
assets.
57. On Dec. 28, 2023, an entity commits itself to purchase c. An entity shall apply the general approach in
equity securities to be classified as held for trading for computing loss allowance on derivative assets.
P1,000,000, its fair value on commitment (trade) date. d. A forward contract for the future purchase or
These securities have a fair value of P1,002,000 and delivery of a commodity or other non-financial item
P1,005,000 on Dec. 31, 2023 (the entity's financial (e.g., gold, electricity, or gas) generally is
year-end), and Jan. 5, 2024 (settlement date), accounted for as a derivative if the contract can be
respectively. If the entity applies the settlement date settled net.
accounting method to account for regular-way
purchases, how much should be recognized in its 2023 62. Which statement is correct regarding accounting for
profit or loss related to these securities? financial instruments in accordance with PFRS 9?
a. Nil c. P3,000 a. PFRS 9 distinguishes between the measurement
b. P2,000 d. P4,000 methods by reviewing the business model of each
entity and the risks and rewards of the transaction.

Page 7 of 8 www.teamprtc.com.ph FAR.1stPB5.23


TEAM PRTC

b. Reclassification adjustments arise on disposal of • Purchased real estate for P220,000 cash which was
investments in equity instruments classified as fair borrowed from a bank.
value through other comprehensive income. • Sold investment in shares designated as FA at
c. An entity is not required to separately recognize FVTOCI for P200,000.
interest revenue or impairment gains or losses for a • Paid dividends of P240,000.
financial asset measured at fair value through profit • Issued 500 ordinary shares for P100,000.
or loss. • Purchased machinery and equipment for P50,000
d. When an entity uses settlement date accounting for cash.
an asset that is subsequently measured at • Paid P180,000 toward a bank loan.
amortized cost, the asset is recognized initially at its • Reduced accounts receivable by P40,000.
fair value on the settlement date. • Increased accounts payable P80,000.
Marcum's net cash used in investing activities for the
63. Which of the following financial assets are initially
current year was
measured at fair value plus transaction costs?
a. P70,000 c. P270,000
a. Derivatives.
b. P20,000 d. P150,000
b. Those that the entity intends to sell immediately or
in the near term.
69. An entity owns a royalty interest in an oil well. The
c. Those that the entity upon initial recognition
contract stipulates that the entity will receive royalty
designates as at fair value through profit or loss.
payments semiannually on Jan. 31 and July 31. The Jan.
d. None of these.
31, payments will be for 20% of the oil sold to jobbers
between the previous June 1 and Nov. 30, and the July
64. An entity’s statement of profit or loss for the year ended
31, payment will be for oil sold between the previous
Dec. 31, 2023 showed a net profit of P83,600. It was
Dec. 1 and May 31. Royalty receipts for 2023 amounted
later found that P18,000 paid for the purchase of a
to P80,000 and P100,000 on Jan. 31 and July 31,
motor van had been debited to the motor expenses
respectively. On Dec. 31, 2022, accrued royalty income
account. It is the entity’s policy to depreciate motor
receivable amounted to P15,000. Production reports
vans at 25% per year on the straight-line basis, with a
show the following oil sales:
full year’s charge in the year of acquisition.
June 1, 2022 – Nov. 30, 2022 P400,000
What would the net profit be after adjusting for this
Dec. 1, 2022 – May 31, 2023 500,000
error?
June 1, 2023 – Nov. 30, 2023 425,000
a. P79,100 c. P101,600
Dec. 1, 2023 – Dec. 31, 2023 70,000
b. P97,100 d. P106,100
What amount should Lorraine report as royalty income
65. The following may be classified as either current or for 2023?
noncurrent, except a. P179,000 c. P184,000
a. Biological assets b. P180,000 d. P194,000
b. Financial assets at fair value through OCI
c. Financial assets at amortized cost 70. An entity had total assets of P40,000, total liabilities of
d. Investment in associate P20,000, and total contributed capital of P8,000 at the
beginning of the year. For the year, the corporation
earned net income of P50,000 and paid cash dividends
Use the following information for the next two questions. of P10,000. At the end of the year, the company had
total assets of P80,000 and its total contributed capital
The accounts and balances shown below were gathered
remained at P8,000. At the end of the year, the
from an entity’s trial balance at the end of the reporting
corporation had total liabilities of:
period.
a. P 0 c. P28,000
Wages Payable P 25,600 b. P20,000 d. P32,000
Cash 17,700
Mortgage Payable 151,600
Dividends Payable 14,000 Thank you for participating in Team PRTC
Prepaid Rent 13,600 Nationwide Online Open First Pre-Board
Inventory 81,800 Examination.
Sinking Fund Assets 52,400
Short-Term Investments 15,200
Premium on Bonds Payable 4,600
Stock Investment - Associate 102,400
Taxes Payable 22,800
Accounts Payable 24,800
Accounts Receivable 36,600

66. The amount that should be reported as current assets


on the entity’s statement of financial position is
a. P151,300 c. P217,300
b. P164,900 d. P267,300

67. The amount that should be reported as current liabilities


on the entity’s statement of financial position is
a. P73,200 c. P 91,800
b. P87,200 d. P238,800

68. Marcum Corp.'s transactions for the current year


included the following:

Page 8 of 8 www.teamprtc.com.ph FAR.1stPB5.23


Excel Professional Services Inc.
Management Firm of Professional Review and Training Center (PRTC)
Online • Manila • Cavite • Laguna • Cebu • Cagayan De Oro • Davao
Since 1977

Advanced Financial Accounting


and Reporting (AFAR) CPA Review DE LEON/DE LEON/ALENTON
FIRST PRE-BOARD EXAMINATION February 19, 20 & 21, 2023

Multiple Choice. Select the letter that corresponds to the or cost-recovery (point in time/time of completion)
best answer. This examination consists of 70 items and the method is employed.
exam is good for three (3) hours. Good luck! b. recognized in the current period under the
percentage-of-completion (over time) method. but
1. Salaries to partners typically should be the cost-recovery (point In time/time of
a. A device for sharing net income completion) method defers recognition of the loss
b. An operating expense of the partnership to the time when the contract is completed.
c. Drawings by the partners c. recognized in the current period under the cost-
d. Reduction for the partners’ capital account balances recovery (point in time/time of completion)
method. but the percentage-of-completion (over
2. Initially, a partner’s interest in a partnership is generally time) method defers the loss until the contract Is
equal to completed.
a. The sum of the fair values of the assets contributed d. deferred and recognized when the contract is
to the firm, increased by the liabilities of the completed, regardless of whether the percentage-
partners assumed by the partnership of-completion (over time) or cost-recovery (point
b. Total fair market value of assets contributed less in time/time of completion) method is employed
liabilities, at book value, to be assumed by the
partnership 8. State the correct sequence of the following steps of
c. Total fair market value of assets contributed revenue recognition under PFRS 15.
d. Net assets contributed at fair market value I. Determine the transaction price
II. Recognize revenue when (or as) the entity
3. Partnership net income is defined as satisfies a performance obligation
a. the interest allocation to the partners, based on III. Identify the performance obligations in the
weighted average invested capital contract
b. partnership income after deducting partner salaries IV. Allocate the transaction price to the
and interest. performance obligations in the contract
c. partnership income after deducting partner V. Identify the contract with the customer
salaries. a. V, IV, II, I, III c. V, III, I, IV, II
d. partnership income before deducting salaries and b. V, I, IV, III, II d. V, I, III, IV, II
interest.
9. Which of the following correctly relates to ‘Step 2’ in the
4. If the total debits in the statement of realization and recognition of revenue under PFRS 15?
liquidation exceeds the total credits, there is a. The entity shall assess the customer’s ability and
a. Net loss for the period c. Either A or B intention to pay the consideration in the contract
b. Net gain for the period d. None of these when they become due.
b. The entity shall determine the transaction price and
5. It is the initial report prepared at the start of the shall consider whether the transaction price
liquidating process. includes, among other things, a variable
a. Cooling-off statement consideration or significant financing.
b. Statement of break up c. The entity shall treat each promise to transfer a
c. Statement of love affairs distinct good or service as a performance
d. Statement of affairs obligation.
d. The entity shall recognize revenue when (or as) a
6. Which of the following is not true about revenue performance obligation is satisfied.
recognition with respect to long-term construction
contracts? 10. A promise to grant a license is most likely to be distinct
a. Long-term construction contract s often are viewed if
as having a single performance obligation, because a. the license is integral to the functionality of a
goods and services fail the "separately identifiable tangible good.
" criterion. b. the customer can benefit from the license only in
b. Long-term construction contracts often satisfy the conjunction with a related service.
criteria for recognizing revenue over time. c. the performance obligation is satisfied over time.
c. Long-term construction contracts require d. the customer can benefit from the license on its own
accounting for construction in progress as well as and the license is separately identifiable.
billings to customers.
d. Long-term construction contracts typically include 11. If the promise to transfer a license is distinct,
multiple performance obligations because of all the a. the entity shall treat all the promises in the contract
different types of goods and services included for as a single performance obligation.
each project b. the entity shall determine whether the
performance .obligation is satisfied over time or at
7. Cost estimates on a long-term contract may indicate a point in time using the general principles' of
that a loss will result on completion of the entire PFRS 15.
contract. In this case, the entire expected loss should c. the entity shall determine the nature of the grant
be of license as either "right to access" or "right to
a. recognized in the current period. regardless of use."
whether the percentage-of completion {over time) d. B and C

Page 1 of 7 www.teamprtc.com.ph AFA.1stPB5.23


TEAM PRTC

12. An entity enters into a contract with a customer to 17. Under PFRS3 Business Combination, goodwill is
license (for a period of three years) intellectual property computed as:
related to the design and production processes for a a. Cost of investments less subsidiary's fair value at
good. The contract also specifies that the customer will the beginning of the year.
obtain any updates to that intellectual property for new b. Cost of investments less subsidiary's book value at
designs or production processes that may be developed the acquisition date.
by the entity. The updates are essential to the c. Cost of investments less subsidiary's fair value at
customer's ability to use the license because the acquisition date.
customer operates in an industry in which technologies d. Cost of investment less subsidiary's book value of at
change rapidly. The entity does not sell the updates the beginning of the year
separately and the customer does not have the option
to purchase the license without the updates. Which of 18. Which of the following Is not a true statement with
the following statements is incorrect? regard to a statutory merger?
a. The promises to grant the license and to provide the a. One entity continues to exist
updates are two separate performance obligations. b. One entity ceases to exist
b. The license and the updates are accounted for c. The name of the new entity is not the same as either
together as a single performance obligation. of the entities
c. The general principles are applied to determine how d. All of the above are true statements with regard to
the performance obligation is satisfied. a statutory merger
d. The single performance obligation is satisfied over
time. 19. Consolidated Net Income attributable to the owners of
the parent is computed as
13. Which of the following would most likely not be a. Net income of the group less non-controlling
considered as a separate performance obligation in interest net income
relation to a franchise agreement? b. Parent net income from its own operation plus share
a. grant of license to use the franchisor's trade name of parent in the subsidiaries net income
b. transfer of equipment to be used in the franchisee's c. Consolidated net Income attributable to owners of
business the parent plus net income attributable to NCI
c. franchisor's promise to undertake activities to d. Parent separate net income plus NCI net income
support the franchise
d. all of the these are separate performance 20. Goodwill is attributed to both the owners of the parent
obligations and non-controlling interests (NCI) if
a. the NCI is measured at 'proportionate share.
14. Which of the following would most likely not be b. the NCI is measured at 'fair value'.
considered as a separate performance obligation in c. in both a and b
relation to a franchise agreement? d. the goodwill is big
a. grant of license to use the franchisor's trade name
b. transfer of equipment to be used in the franchisee's The balance sheet of Piedmont Enterprises and Skelton
business Company at December 31, 2021 are summarized as follows:
c. franchisor's promise to undertake activities to
support the franchise Piedmont Skelton
d. all of the these are separate performance Assets P5,000,000 P 2,000,000
obligations Liabilities P1,500,000 P 500,000
Capital stock , P40 par 2,500,000
15. Statement 1 (S1): The balance of the Allowance for Capital stock, P25 par 1,000,000
Overvaluation of Inventories: Branch ledger account is Retained earnings 1,000,000 500,000
deducted from the balance of the Investment in Branch
account in the separate balance sheet of the home At the date of acquisition, Skelton’s assets are understated
office. while its liabilities are fairly valued.
Statement 2 (S2]: If the home office bills shipments of
merchandise to the branch ai 25% above home office On January 1, 2022, Piedmont purchased 80% of Skelton
cost and the adjusted balance of the Allowance for Company’s outstanding shares for P2,000,000 when the fair
Overvaluation of Inventories: Branch ledger account is value of Skelton’s net assets was P2,200,000. Piedmont
P20,400, the amount of branch inventories at billed issued 10,000 unissued shares in consideration of the
prices is P81,600. acquisition. Piedmont is to assign an amount to the non-
a. S1 - True; S2 – True c. S1 - False: S2 - True controlling interests at the date of acquisition based on the
b. S1 - True; S2 – False d. S1 - False: S2 - False total fair value of Skelton’s outstanding shares.

16. Statement 1 (S1): A home office records shipments to 21. How much is the consolidated assets at the date of
its branch at billing prices and adjusts the loading acquisition?
account (deferred profit) at year-end. When this a. P9,000,000 c. P8,000,000
approach is used, the loading account during the period b. P9,700,000 d. P 8,700,000
will always be zero.
Statement 2 (S2): If a "loading" account is used, the 22. How much is the consolidated liability at the date of
"shipments to branch" account on the home office acquisition?
books is created for the actual cost of shipments made a. P2,000,000 c. P1,800,000
to the branch whereas the "shipments from the home b. P1,500,000 d. P 500,000
office" on the branch's books includes any initial
unrealized profit 23. How much is the stockholders’ equity in the
a. S1 - True; S2 – True c. S1 - False: S2 - True consolidated balance at January 1, 2022?
b. S1 - True; S2 – False d. S1 - False: S2 - False a. P7,000,000 c. P6,000,000
b. P5,500,000 d. P6,700,000

Page 2 of 7 www.teamprtc.com.ph AFAR.1stPB5.23


TEAM PRTC

28. Assuming DLSU is not an SME, determine the amount


24. Assume the amount assigned to the non-controlling of goodwill / ( income from acquisition) that will result
interest at the date of acquisition is based on the total from the combination.
fair value of identifiable net assets at that date, calculate a. P 20,000 c. P 40,000
the amount of goodwill recognized at January 1, 2022. b. (P 40,000) d. P 95,000
a. P300,000 c. P 20,000
b. P280,000 d. P240,000 29. Assuming DLSU’s Retained earnings at January 1, 2018
was P500,000, at what amount will the Retained
The Carl Company will issue P10 par value common stock Earnings be shown on the balance at this date?
for the net assets of PBA Company. The fair market value DLSU is not an SME DLSU is an SME
per share of Carl’s common stock is P40. The following is
a. P 445,000 P 485,000
the list of accounts of PBA Company on the date of the
acquisition. b. P 365,000 P 445,000
c. P 485,000 P 405,000
Book Value Fair Market Value d. P 405,000 P 485,000
Current assets P280,000 P 320,000
Plant assets (net) 680,000 1,280,000 BAHAY-PARE CORPORATION purchases all the outstanding
Liabilities 320,000 shares of SINAG-TALA COMPANY on January 2, 2022 for
Common stock 64,000 P385,000 cash. On this date the stockholders equity of
Additional paid-in capital 256,000 SINAG-TALA is as follows:
Retained earnings 320,000
Share capital, P10 par P175,000
25. To have an income from acquisition of P120,000, the Paid-in capital in excess of par 87,500
number of shares to be issued by Carl Company should Retained earnings 175,000
be
a. 30,000 shares c. 29,000 shares Any excess of the fair value of net assets over the fair value
b. 30,400 shares d. 35,000 shares of the investment is attributable to SINAG-TALA’s building
which is currently overstated in its books. All other net asset
26. Same data as in previous number, to have a goodwill of items of the acquired company are fairly valued at the
P 120,000, the number of shares to be issued by Carl acquisition date. The building has an estimated life of 10
Company should be years from January 2, 2022 without salvage value.
a. 30,000 shares c. 29,000 shares
b. 30,400 shares d. 35,000 shares The condensed trial balances of the affiliated companies on
December 31, 2022 appear as follows:

The following balance sheet was prepared for ACADEME BAHAY-PARE SINAG-
COMPANY just before DLSU CORPORATION acquired its TALA
entire net assets on January 1, 2018. Current assets P 420,000 P 302,750
Land 210,000 210,000
Particulars Book Value Fair Value Building (net) 1,050,000 283,500
Cash P 10,000 P 10,000 Investment in SINAG- 385,000 -
Accounts receivable 40,000 40,000 TALA
Inventory 100,000 145,000 Current liabilities (708,750) (367,500)
Plant assets 300,000 350,000 Ordinary shares, P3 par (525,000) -
Goodwill 50,000 - Share capital, P10 par - (175,000)
P500,000 P545,000 Paid-in capital in (315,000) ( 87,500)
Accounts payable P140,000 P140,000 excess of par
Bonds payable 60,000 65,000 Retained earnings, Jan. (446,250) (175,000)
Ordinary shares 200,000 2, 2022
Share premium 20,000 Sales (367,500) ( 70,000)
Retained earnings 80,000 Cost of goods sold 210,000 61,250
P500,000 Operating expenses 78,750 17,500
Dividends declared 8,750 -
Totals -- --
DLSU issued 10,000 shares of stocks with a par value per
share of P5 and a fair value of P30. Additional cash
30. Compute the consolidated net income for 2022.
payments made by DLSU in completing the acquisition
a. P75,520 c. P72,550
were:
b. P70,525 d. P75,250
Broker’s fee paid to firm that located P10,000
31. Compute the consolidated Retained Earnings at
ACADEME COMPANY
December 31, 2022.
Cost to issue and register the shares 40,000
a. P517,250 c. P515,270
Professional fees paid to accountants 30,000
b. P525,170 d. P512,750
Professional fees paid to lawyers 25,000
Professional fees paid to official valuers 15,000 The following information was taken from the books of
Indirect acquisition costs 15,000 MAYON COMPANY and its Naga City branch on December
31, 2022, before adjusting entries were recorded.
27. Assuming DLSU is an SME, calculate the goodwill /
(income from acquisition) that will result from the Branch books
combination. Sales P300,000
a. P 20,000 c. P 40,000 Inventory, January 1 19,000
b. (P 40,000) d. P 95,000 Purchases 20,000
Shipment from Home Office 180,000

Page 3 of 7 www.teamprtc.com.ph AFAR.1stPB5.23


TEAM PRTC

Expenses 80,000 activities. An assessment of their participation shows they


Home Office books have rights over assets and obligations over liabilities. They
Sales P400,000 have equal shares in interest.
Inventory, January 1 40,000
Purchases 210,000 On July 1, 2022, Amer sells machinery to the joint
Shipment to Branch 150,000 undertaking for P102,400. The cost to Amer of the
Expenses 210,000 machinery transferred is P40,960. The machinery had an
Allowance for overvaluation of branch 31,500 estimated remaining useful life of 5 years at that date.
inventory, December 31
There are no merchandise shipments in transit as at the 38. At what amount will Amer show the machinery, net at
year-end. The ending inventories are: its balance sheet at July 1, 2022?
a. P13,653 c. P17,067
Home Office (all from outside suppliers) P50,000
b. P34,133 d. P17,076
Branch office (40% from outside suppliers, 40,000
the rest from home office)
39. At what amount will Amer show the machinery, net at
its balance sheet at December 31, 2022?
32. How much is the overstatement of the branch’s 2022
cost of sales per home office cost?
a. P27,306 c. P34,133
a. P 12,500 c. P 27,000 b. P10,922 d. P12,288
b. P 13,500 d. P 27,500
40. At what amount will Brad show the machinery in its
33. Calculate the company’s 2022 net income. balance sheet at December 31, 2022?
a. P 140,000 c. P 181,000 a. P27,320 c. P37,200
b. (160,000) d. P 208,500 b. P30,720 d. P32,700

On January 1, 2020, SME Voltex 5 Company has a 30%


The Dasmarinas Corporation operates a branch in Calamba equity of Takuza 4 Enterprises for P92,800. The latter
City. The home office ships merchandise to the branch at company is a joint venture undertaking. Transaction costs
more than cost. Selected information Selected information of 3% of the purchase price of the shares were incurred by
from the December 31, 2022 trial balances are as follows: SME Voltex 5 Company.

Home Branch On December 31, 2010, Takuza 4 declared and paid


Office Office dividends of P24,000 and reported a profit of P67,200
Books Books Published price quotations do not exist for Takuza shares
Sales P600,000 P300,000 but appropriate valuation techniques determined the fair
Shipment to Branch 200,000 value of the investment at P104,000. Costs to sell are
Purchases 350,000 - estimated at P5,200.
Shipment from Home office 230,000
Inventory, January 1 100,000 40,000 41. What is the amount of the Investment in JV to be
Allowance for overvaluation of 58,000 recognized by Voltex 5 in its 2020 balance sheet using
branch inventory the fair value method?
Expenses 120,000 50,000 a. P 94,640 c. P 65,520
Inventory at December 31, 2022 b. P104,000 d. P 92,800
Home office P30,000
Branch office P40,000 42. What is the amount of Investment in JV to be recognized
by Voltex 5 in its 2020 balance sheet under the equity
Merchandise shipment in transit at December 31, 2022 is method.
P20,000 at billed price. a. P 108,544 c. P98,800
b. P 180,445 d. P89,800
34. The net income reported by the home office for its
2022 operation is: On March 31, 2022, Emng, Bobby, and Ramil formed the
a. P260,000 c. P 20,000 POGI Partnership to operate a CPA review center. The
b. P 38,000 d. P200,060 following is a list of their contributions at that date:

In ‘000 Emong Bobby Ramil


35. The net income reported by the branch for its 2022
operation is Book Fair Book Fair Book Fair
Value value Value value Value Value
a. P20,000 c. P260,000
Cash P132 P132 P100 P100 P120 P120
b. P200,060 d. P 38,000
Inventor 80 75
y
36. How much is the overstatement of the cost of sale in Land 150 188
the branch 2022 income statement resulting from the Equipme
home office billing policy? nt, net ____ ____ _____ _____ 90 90
a. P46,000 c. P20,000
Totals P282 P320 P180 P175 P210 P210
b. P38,000 d. P 0
Bobby has an accounts payable of P50,000 on the inventory
37. How much net income was reported in the company’s
and Ramil has a mortgage payable of P60,000 on the
2022 income statement?
equipment. The partners have agreed to assume only the
a. P362,000 c. P236,000 mortgage payable but not the accounts payable. They
b. P263,000 d. P326,000 further agreed for the capital ratio to be 50%, 20%, and
30% to Emong, Bobby, and Ramil, respectively.:
Amer Company, Brad Company, and Cris Enterprises are
participants in a joint undertaking bound by a contractual
agreement for the sharing of control over its relevant

Page 4 of 7 www.teamprtc.com.ph AFAR.1stPB5.23


TEAM PRTC

The partnership starts operation on April 1, 2022 and on value through the non-cash assets prior to Clara’s
December 31, 2022 reported a net income of P305,400. admission.

The following is the profit and loss agreement among the 47. How much will be the capital balances of Alma and
partners Bella after the admission of Clara?
• 10% interest to each partner’s beginning capital a. P 150,000 and P150,000
• Salaries of P30,000 per quarter will be given to Emong b. P 210,000 and P190,000
and Ramil c. P210,000 and P210,000
• Bonus of 10% of net income after interest, salaries, and d. P190,000 and P19,000
bonus will be given to Emong.
• Residual profit/(loss) will be divided equally. 48. The entry to record the admission of Clara will not
include
43. How much is the net asset contribution of Ramil? a. A debit to Cash of P400,000
a. P210,000 c. P175,000 b. A debit to Alma, capital of P210,000
b. P150,000 d. P125,000 c. A debit to Bella, capital of P190,000
d. A credit to Clara, capital of P400,000
44. What is the beginning capital of Partner Bobby?
a. P352,500 c. P129,000 The balance sheet of Abby, Blanche, and Celia partnership
b. P211,500 d. P141,000 on January 1, 2019, the date of partnership dissolution, was
as follows:
On December 31, 2019, the balance sheet for the XYZ
Partnership follows: Cash P 4,000 Liabilities P 8,000
Other 26,000 Abby, loan 1,000
Cash P 10,000 Accounts payable P 17,500 assets
Accounts 15,000 Loan from Zoilo 12,500 Celia, loan 2,000 Abby, capital (20 2,000
receivable % P/L)
Inventory 35,000 Xander, capital 35,000 Blanche, capital(40 9,000
(20%) % P/L)
Plant 30,000 Ysabel, capital 25,000 ________ Celia, capital (40 12,000
assets, net (20%) % P/L)
Loan to 15,000 Zoilo, 15,000 P 32,000 P 32,000
Xander capital(60%)
Total P105,000 Total P105,000 In January, other assets with a book value of P16,000 were
assets liability/equity sold for P10,000.

The percentages shown are for the residual profit and loss 49. How much will each partner receive from the cash
sharing ratios. The partners dissolved the partnership on distribution after the liabilities had been paid.
January 1, 2020 and began the liquidation process. During a. Abby, P1,200; Blanche, P1,800; and Celia, P3,000
January the following events occurred: b. Abby, P 0; Blanche, P2,500; and Celia P3,500
c. Abby P1,800; Blanche , P1,800; and Celia, P2,400
• Receivables of P7,500 were collected. d. Abby, P 0; Blanche, P2,000; and Celia, P 4,000
• All inventory was sold for P10,000.
• All available cash was distributed on January 31, 2020, 50. If the partners have retained available cash of P400 for
except for P5,000 that was set aside for contingent future liquidation expenses, after the liabilities have
expenses. been paid, how much will Blanche receive from the
cash distribution?
45. How much cash would Ysabel receive from the cash a. P1,800 c. P2,500
that is available for distribution on January 31 b. P2,000 d. P2,300
a. P0 c. P 5,000
b. P3,000 d. P 1,000 The following data were taken from the Statement of Affairs
of Greenfield Corporation.
46. How much cash would Xavier receive from the cash
that is available for distribution on January 31 Pledged Assets : BCV ERV
a. P 5,000 c. P 3,000 Plant, property, and equipment P72,000 P60,000
b. P0 d. P1,000 (PPE)
Merchandise inventory 59,200 41,600
Alma and Bella formed a partnership in the Philippines,
Free assets 56,000 32,000
which uses PFRS based on IASB accounting principles. The
two partners agree on a profit and loss ratio of 60% and Total assets P187200 P133,600
40% to Alma and Bella, respectively. At a later date, the Secured liabilities
partners agree to admit Clara into the partnership for a Bonds payable (secured by P24,000
50%interestin capital and in earnings. PPE)
Notes payable (secured by 48,000
Capital accounts of the partners immediately before the merchandise inventory)
admission of Clara are: Alma, P300,000 and Bella,
Unsecured liabilities:
P300,000.
Clara invested P400,000 for the partnership interest and Taxes P 3,000
that this is a fair price for the share of partnership interest Salaries and wages 2,600 5,600
to be acquired. Clara paid the money directly to Alma and Accounts payable 89,600
Bella for 50% each of their existing interests. The partners
have decided to revalue partnership interest to current fair

Page 5 of 7 www.teamprtc.com.ph AFAR.1stPB5.23


TEAM PRTC

51. What is the estimated amount the holders of the notes


payable will receive in the event of liquidation? Best Ever Constructions uses the cost-to-cost method in
a. P52,700 c. P56,200 measuring the progress of satisfying performance
b. P45,760 d. P57,000 obligation in contracts with customers. During 2022, a single
long-tem contract was begun for a contracted price of
52. What is the estimated amount the unsecured creditors P2,135,000. Information on the project follows:
with priority will receive in the event of liquidation?
a. P5,600 c. P7,500 2022 2023
b. P6,000 d. P6,200 Revenue earned to date P 427,000 P1,281,000
Progress billings to date 350,000 1,470,000
53. What is the estimated amount holders of the accounts Cost incurred to dat 367,500 1,039,500
payable will receive upon liquidation? Collections to date 320,000 1,050,000
a. P58,240 c. P52,480
b. P54,840 d. P89,600 58. What is the gross profit recognized from this long-
term contract in
54. Using the data provided EXCEPT that the accounts 2022 2023 2022 2023
payable is P49,600 (instead of P89,600), compute the a. P77,000 P798.000 c. P59,500 P 175,000
estimated amount the stockholders will receive upon b. P77,000 P350,000 d. P59,500 P 182,000
liquidation.
a. P10,400 c. P14,000 CCC Construction Company started work on three job sites
b. P 0 d. P 6,400 with different customers during 2020. Data relating to the
three jobs are given below (In ‘000):
Amounts related to the statement of affairs of Distressed
Company as of April 30, 2019 follow: Site Contract Costs Costs to Progress Collections
Price Incurred Complete Billings
Abra P500 P375 P0 P500 P400
Assets pledged for fully secured P 80,000
liabilities Bohol 700 100 400 100 80
Assets pledged for partially secured 50,000 Cebu 250 100 100 130 120
liabilities
Free assets 272,000 59. What amount of gross profit should be recorded for the
Fully secured liabilities 60,000 current year if the cost recovery method is used for all
Partially secured liabilities 80,000 contracts?
Unsecured liabilities with priority 40,000 a. P100,000 c. P240,000
Unsecured liabilities without priority 330,000 b. P125,000 d. P375,000

55. Calculate the expected amount recoverable by partially 60. What amount of gross profit would be reported for the
secured creditors in the event of liquidation. current year if the percentage of completion method is
a. P71,000 c. P69,500 used for all contracts?
b. P50,000 d. P80,000 a. P 65,000 c. P215,000
b. P190,000 d. P240,000
Home office bills its branch for merchandise shipments at
30% above cost. 61. Aside from uncollected contract receivable, at what
The following are some of the account balances on the books amounts should (1) Bohol and (2) Cebu be shown in
of home office and its branch as of December 31, 20X0: the balance sheet at the year-end, respectively, under
the % of completion method?
Home Office Branch a. (1) P40,000 Contract asset and (2) P5,000
Books Books contract liability
Inventory, January 1 35,000 101,500 b. (1) P20,000 Contract asset and (2) P10,000
Shipments from Home Office 263,900 Contract Asset
Purchases 1,575,000 350,000 c. (1) P40,000 contract liability and (2) P10,000
Shipments to Branch 253,750 contract asset
Branch Inventory Allowance 91,875 d. (1) P40,000 contract liability and (2) P5,000
Sales 2,100,000 1,260,000 contract asset
Operating Expenses 507,500 192,500

Per physical count, the ending inventory of the branch is


P73,500 including goods from outside purchases of
P48,475; the ending inventory of the home office is
P210,000.

56. What is the total ending inventory to be shown on the


combined financial statements?
a. P118,475
b. P277,725
c. P328,475
d. P280,000
.
57. What is the combined net income for the year?
a. P957,950
b. P871,850
c. P891,975
d. P942,725

Page 6 of 7 www.teamprtc.com.ph AFAR.1stPB5.23


TEAM PRTC

BRIGADIER CONSTRUCTIONS, INC. had been working on a. a debit to consignment-Out of P7,935


three different projects during 2023, which had been started b. a credit to Consignment profit of P8,255
in different years. The following data pertain to the jobs at c. a credit to Consignment-Out of P9,375
the end of 2023. The company uses the percentage of d. a debit to Consignment-In of P P37,500
completion method in accounting for long-term projects.
Joey Muffler sells franchise arrangements throughout Luzon
Job Const Contract Costs To- and Visayas. Under a franchise agreement, Joey receives
Period price date P600,000 in exchange for satisfying the following
performance obligations:
1 2023 520,000 330,000
• franchisees have a five-year right to Operate as a Joe
Muffler retail establishment in an exclusive sales
2 2022 5,529,600 1,843,200 territory.
• franchisees receive initial training and certification as a
2023 3,446,784 Muffler Mechanic, and
• franchisees receive a Joey Muffler. building and
necessary equipment.
3 2021 8,000,000 1,024,000

2022 3,993,600 The stand-alone selling price of the initial training and
certification is P15,000, and P450,000 for the building and
2023 6,473,600 equipment. Joey received P75,000 on July 1, 20x6, from
Althea and accepted a note receivable for the rest of the
franchise price. Joey will construct and equip Altheas'
Job Estimated Billings To- Collections
building and train and certify Althea by September 1, and
Costs To Date To-Date
Altheas' five-year right to operate as a Joey Muffler
Complete
establishment will commence on September 1 as well.
1 --- 520,000 440,000
Franchisee has the right to access on the franchise
agreement.
2 P3,072,000 1,935,360 1,843,200
2,297,856 3,686,400 3,512,080 68. What amount would Joey assign as the stand-alone
selling price of the five year right to operate as a Joey
3 4,096,000 1,500,000 1,310,000 Muffler retail establishment?
a. P135,000 c. P585,000
2,246,400 5,250,000 5,000,000
b. P150,000 d. P600,000
--- 8,000,000 7,800,000
69. How much revenue would Joey recognize in the year
62. The gross profit recognized in 2021 was ended December 31, 20x6, with respect to its franchise
a. P675,000 c. P756,000 arrangement with Althea? (Ignore any interest on the
b. P657,000 d. P576,000 note receivable.)
a. P9,000 c. P465,000
63. The total gross profit for all the projects to be b. P450,000 d. P474,000
recognized in 2023 will be
a. P144,650 c. P146,450 70. Ronella Ocampo sells hairstyling franchises. Ronella
b. P144,560 d. P145,460 Ocampo receives P50,000 from a new franchisee for
providing equipment and furnishings that have a stand-
Under a consignment arrangement, CONSIGNOR COMPANY alone selling price of P50,000. Ronella Ocampo also
ships 24 units of special electronic gadgets called Tsamba receives P30,000 per year for use of, the Ronella
Bebe to CONSIGNEE ENTERPRISES. The cost per unit to Ocampo name and for ongoing consulting services
Consignor, Inc. is P1,500 and the goods will be sold at (starting on the date franchise is purchased). Carlos
P2,400 each for a 10% commissions on gross sales. Freight became a Ronella Ocampo franchisee on July 1, 20x6
charges paid by the consignor on the shipments was P1,500. had completed training and was open for business. How
At the end of 30 days, the consignee rendered an Account much revenue in 20x6 will Ronella Ocampo recognize
Sales for 18 units sold and reimbursable charges for: for its arrangements with Carlos?
Delivery expenses, P1,000 and Advertising, P1,500, among a. Zero c. P65,000
others. The consignor uses the Consignment-Out account in b. P10,000 d. P70,000
accounting for the consignments. On the other hand, the
Consignee uses the Consignment-In account. Thank you for participating in Team PRTC
Nationwide Online Open First Pre-Board
64. How much is the net remittance by the consignee? Examination.
a. P36,380 c. P33,680
b. P36,830 d. P30,638

65. How much is the net profit of the consignor?


a. P 8,705 c. P 8,255
b. P 5,555 d. P 2,513

66. How much is the adjusted balance of the Consignment


Out account after recognition of the net profit?
a. P7,935 c. P 5,973
b. P9,375 d. P 3,957

67. The reclassification entry to be recorded by the


consignor to recognize the deferred cost in its balance
sheet will include

Page 7 of 7 www.teamprtc.com.ph AFAR.1stPB5.23

You might also like